Good Doctor 3

Pataasin ang iyong marka sa homework at exams ngayon gamit ang Quizwiz!

A nurse is providing care for a postpartum client. Which condition increases this client's risk for a postpartum hemorrhage? hypertension uterine infection placenta previa severe pain

placenta previa Explanation: The client with placenta previa is at greatest risk for postpartum hemorrhage. In placenta previa, the lower uterine segment doesn't contract as well as the fundal part of the uterus; therefore, more bleeding occurs. Hypertension, severe pain, and uterine infection don't increase the client's risk for postpartum hemorrhage.

A client with an uncomplicated term pregnancy arrives at the labor-and-delivery unit in early labor saying that she thinks her water has broken. What is the nurse's best action? Prepare the woman for birth. Ask what time this happened and note the color, amount, and odor of the fluid. Immediately contact the provider. Collect a sample of the fluid for microbial analysis.

Ask what time this happened and note the color, amount, and odor of the fluid. Explanation: Gather more information. Noting the color, amount, and odor of the fluid, as well as the time of rupture, will help guide the nurse in the next action. There's no need to immediately call the client's provider or prepare this client for birth if the fluid is clear and birth isn't imminent. Rupture of membranes isn't unusual in the early stages of labor. Fluid collection for microbial analysis is not routine if there's no concern for infection.

While assessing a multigravid client at 10 weeks' gestation, the nurse notes a purplish color to the vagina and cervix. The nurse documents this as what finding? Goodell's sign Chadwick's sign Hegar's sign melasma

Chadwick's sign Explanation: A purplish blue discoloration of the vagina and cervix is termed Chadwick's sign; it is caused by increased vascularity of the vagina during pregnancy and is considered a probable sign of pregnancy. Goodell's sign, also considered a probable sign of pregnancy, refers to a softening of the cervix during pregnancy. Hegar's sign, also a probable sign of pregnancy, refers to a softening of the lower uterine segment. Melasma, the mask of pregnancy, refers to the pigmentation of the skin on the face during pregnancy. Melasma is considered a presumptive sign of pregnancy.

While assisting the physician with an amniocentesis on a multigravid client at 38 weeks' gestation, the nurse observes that the fluid is very cloudy and thick. The nurse interprets this finding as indicating which of the following? Intrauterine infection. Fetal meconium staining. Erythroblastosis fetalis. Normal amniotic fluid.

Intrauterine infection. Explanation: Thick, cloudy amniotic fluid indicates an intrauterine infection. Typically, the client has a fever, lethargy, and malaise. Greenish-colored amniotic fluid is associated with meconium staining. A strong yellowish color is associated with erythroblastosis fetalis because of the presence of bilirubin and hemolyzed red blood cells. The normal color of amniotic fluid is clear or with a very slight yellow tint later in pregnancy.

Which intervention should the nurse anticipate using when caring for a term neonate diagnosed with transient tachypnea at 2 hours after birth? Monitor the neonate's color and cry every 4 hours. Feed the neonate with a bottle every 3 hours. Obtain extracorporeal membrane oxygenation equipment. Provide warm, humidified oxygen in a warm environment.

Provide warm, humidified oxygen in a warm environment. Explanation: Symptoms of transient tachypnea include respirations as high as 150 breaths/minute, retractions, flaring, and cyanosis. Treatment is supportive and includes provision of warm, humidified oxygen in a warm environment. The nurse should continuously monitor the neonate's respirations, color, and behaviors to allow for early detection and prompt intervention should problems arise. Feedings are given by gavage rather than bottle to decrease respiratory stress. Obtaining extracorporeal membrane oxygenation equipment is not necessary but may be used for the neonate diagnosed with meconium aspiration syndrome.

A client is receiving oxytocin to induce labor. Which assessment findings indicate common adverse reactions? Select all that apply. blood pressure increased from baseline uterine tetany jaundice heart rate decreased from baseline variable decelerations

blood pressure increased from baseline uterine tetany variable decelerations Explanation: Common adverse reactions to oxytocin include hypertension, tachycardia, uterine tetany and hypertonicity, and variable decelerations. Oxytocin does not cause jaundice. Tachycardia, not decreased heart rate, is a common adverse reaction.

A nurse is assisting in developing a teaching plan for a client who is about to enter the third trimester of pregnancy. The teaching plan should note that which symptom should be reported immediately? hemorrhoids blurred vision dyspnea on exertion increased vaginal mucus

blurred vision Explanation: During pregnancy, blurred vision may be a danger sign of preeclampsia or eclampsia, complications that require immediate attention because they can cause severe maternal and fetal consequences. Although hemorrhoids may occur during pregnancy, they do not require immediate attention. Dyspnea on exertion and increased vaginal mucus are common discomforts caused by the physiologic changes of pregnancy.

The end of the third stage of labor is marked by what event? the birth of the neonate complete dilation delivery of the placenta transfer of the client to the postpartum bed

delivery of the placenta Explanation: The third stage of labor is marked by the delivery of the placenta. The first stage of labor ends with complete cervical dilation and effacement. The second stage of labor ends with the birth of the neonate. The fourth stage of labor includes the first 2 hours after birth.

A nurse is completing a prenatal assessment on a woman who is 28 weeks pregnant with gestational hypertension. Which findings should be reported to the primary care provider? Select all that apply. dull headache weight gain of 1 lb (500 g) per week blurred vision 1+ urine protein fundal height of 28 centimeters

dull headache blurred vision 1+ urine protein Explanation: The nurse must be alert for any signs and symptoms of superimposed preeclampsia in women with gestational hypertension. Dull headache, blurred vision, and protein in urine are all classic signs of preeclampsia in pregnancy and must be reported to the primary care provider immediately. Weight gain of 1 lb (500 grams) per week is an expected finding. Fundal height of 28 cm is an expected finding.

The nurse is assessing an hour-old newborn. Which observations would the nurse note as being abnormal? Select all that apply. respiratory rate of 56 heart rate of 135 expiratory grunting temperature of 97.4° F (36.3° C) nasal flaring

expiratory grunting temperature of 97.4° F (36.3° C) nasal flaring Explanation: Respiratory rate for a newborn is 30-60 and the heart rate is 120-160. Expiratory grunting and nasal flaring are signs of respiratory distress in the newborn. Temperature for a newborn should be between 97.5° F (36.4° C) and 99° F (37.2° C).

A client makes a routine visit to the prenatal clinic. Although the client is 14 weeks pregnant, the size of her uterus approximates an 18- to 20-week pregnancy. The physician diagnoses gestational trophoblastic disease and orders ultrasonography. The nurse expects ultrasonography to reveal: an empty gestational sac. grapelike clusters. a severely malformed fetus. an extrauterine pregnancy.

grapelike clusters. Explanation: In a client with gestational trophoblastic disease, an ultrasound performed after the third month shows grapelike clusters of transparent vesicles rather than a fetus. The vesicles contain a clear fluid and may involve all or part of the decidual lining of the uterus. Usually no embryo (and therefore no fetus) is present because it has been absorbed. Because there is no fetus, there can be no extrauterine pregnancy. An extrauterine pregnancy occurs with an ectopic pregnancy.

The primary health care provider (HCP) prescribes betamethasone for a 34-year-old multigravid client at 32 weeks' gestation who is experiencing preterm labor. Previously, the client has experienced one infant death due to preterm birth at 28 weeks' gestation. The nurse explains that this drug is given for which reason? to enhance fetal lung maturity to counter the effects of tocolytic therapy to treat chorioamnionitis to decrease neonatal production of surfactant

to enhance fetal lung maturity Explanation: Betamethasone therapy is indicated when the fetal lungs are immature. The fetus must be between 28 and 34 weeks' gestation and birth must be delayed for 24 to 48 hours for the drug to achieve a therapeutic effect. Antibiotics would be used to treat chorioamnionitis. Betamethasone is not an antagonist for tocolytic therapy. It increases, not decreases, the production of neonatal surfactant.

A multigravid client in active labor at term suddenly sits up and says, "I can't breathe! My chest hurts really bad!" The client's skin begins to turn a dusky gray color. After calling for assistance, which action should the nurse take next? Administer oxygen by face mask. Begin cardiopulmonary resuscitation. Administer intravenous oxytocin. Obtain an prescription for intravenous fibrinogen.

Administer oxygen by face mask. Explanation: The client's symptoms are indicative of amniotic fluid embolism, which is a medical emergency. After calling for assistance, the first action should be to administer oxygen by face mask or cannula to ensure adequate oxygenation of mother and fetus. If the client needs cardiopulmonary resuscitation, this can be started once oxygen has been administered. If the client survives, disseminated intravascular coagulation will probably develop, and the client will need intravenous fibrinogen and heparin. Oxytocin, a vasoconstrictor, is not warranted for amniotic fluid embolism.

A client who is 29 weeks pregnant comes to the labor and childbirth unit. She states that she is having contractions every 8 minutes. The client is also 3 cm dilated. Which can the nurse expect to administer? Select all that apply. folic acid a β-2 agonist betamethasone Rohm(D) immune globulin (RhoGAM) intravenous fluids nalbuphine

a β-2 agonist betamethasone intravenous fluids Explanation: The nurse can expect that a β-2 agonist that relaxes smooth muscle will be administered to halt contractions; that betamethasone, a corticosteroid, will be administered to decrease the risk of respiratory distress to the neonate if preterm birth occurs; and that intravenous fluids will be given to expand the intravascular volume and decrease contractions if dehydration is the cause. Folic acid is a mineral recommended throughout pregnancy (especially in the first trimester) to decrease the risk of neural tube defects. RhoGAM is given to Rh-negative clients who have been, or may have been, exposed to Rh-positive fetal blood. Nalbuphine is an opioid analgesic used during labor and birth.

The nurse is preparing to administer erythromycin ophthalmic ointment to a neonate soon after birth. The nurse should explain to the parents that this medication, in addition to preventing blindness caused by gonococcal organisms, also prevents neonatal blindness caused by which organism? beta-hemolytic streptococcus Escherichia coli Chlamydia trachomatis Staphylococcus aureus

Chlamydia trachomatis Explanation: The use of erythromycin ophthalmic ointment prevents blindness from gonococcal organisms and C. trachomatis. This ointment usually is less expensive than tetracycline.Beta-hemolytic streptococcus, E. coli, and S. aureus can cause a generalized infection in the neonate. However, these organisms typically are not responsible for causing neonatal blindness.

A woman who gave birth to her last infant by caesarean birth is admitted to the hospital at term with contractions every 5 minutes. The health care provider (HCP) intends to have her undergo "a trial labor." What does the nurse explain to the client that trial of labor means? Labor will be stimulated with exogenous oxytocin until delivery. The HCP needs more information to determine the presence of true labor. Labor progress will be evaluated continually to determine appropriate progress for a vaginal delivery. Labor will be arrested with tocolytic agents after a 2-hr period even if no fetal distress is noted.

Labor progress will be evaluated continually to determine appropriate progress for a vaginal delivery. Explanation: A trial labor in this context means that the woman is allowed to go into labor, and her progress is assessed by cervical dilation and effacement as well as fetal descent evaluated to determine whether to allow the labor to progress to delivery. If there are indications that labor is not progressing, other means of delivery are considered. Labor stimulation is used cautiously and may not be safe. The presence of contractions every 5 minutes indicates true labor. If fetal distress is noted and an emergency cesarean birth cannot be done immediately, tocolytic agents may be considered to stop contractions.

A pregnant client calls the nurse at 22 weeks gestation to report that she is experiencing some edema of her face and hands, with puffiness in her eyelids in the morning. What is the priority action by the nurse? Explain that this is a normal finding for pregnancy. Suggest that the client recline in a lateral recumbent position. Tell the client to monitor her symptoms for 24 hours. Refer the client to her physician.

Refer the client to her physician. Explanation: With preeclampsia, edema begins to accumulate in the upper part of the body, rather than just the typical ankle edema of pregnancy. With severe preeclampsia, the edema will be present in the woman's hands and face as puffiness and is not responsive to 12 hours of bed rest. A physician needs to be seen to determine appropriate treatment. Other actions are not appropriate at this time.

A client asks, "Can my partner and I still engage in sexual intercourse while I'm pregnant?" What should the nurse tell the client? "Throughout the pregnancy, coitus interruptus is the preferred method for sexual activity." "Although your sexual desire may change, intercourse is safe during an uncomplicated pregnancy." "You should avoid having intercourse until you are at least 16 weeks pregnant." "Refrain from having sexual intercourse when you are in the last trimester."

"Although your sexual desire may change, intercourse is safe during an uncomplicated pregnancy."

After the nurse reinforces the danger signs to report with a gravida 2 client at 32 weeks' gestation with an elevated blood pressure, which client statements would demonstrate understanding of when to call the primary health care provider's (HCP's) office? Select all that apply. "If I feel dizzy when I get up quickly." "If I see any bleeding, even if I have no pain." "If I have a pounding headache that will not go away." "If I notice the veins in my legs getting bigger." "If the leg cramps at night are waking me up." "If the baby seems to be more active than usual."

"If I see any bleeding, even if I have no pain." "If I have a pounding headache that will not go away." "If the baby seems to be more active than usual." Vaginal bleeding with or without pain could signify placenta previa or abruptio placentae. Continuous or pounding headache could indicate an elevated blood pressure, and change in the strength or frequency of fetal movements could indicate that the fetus is in distress. Orthostatic hypotension can occur during pregnancy and can be alleviated by rising slowly. Leg veins may increase in size due to additional pressure from the increasing uterine size, while leg cramps may also occur and can commonly be decreased with calcium supplements.

A 2-day postpartum client tells the nurse that she is experiencing abdominal cramps whenever she breastfeeds her baby. Which is the most appropriate response from the nurse? "Progesterone levels increase after birth of the placenta which triggers milk production and uterine contractions." "Oxytocin is released when the baby sucks which causes the uterus to contract." "The baby is feeding too frequently causing the uterus to be hyper-stimulated." "The cramps will stop after your body has expelled all blood clots from the uterus."

"Oxytocin is released when the baby sucks which causes the uterus to contract." Explanation: Afterpains, which are intermittent cramping of the uterus, tend to be noticed by multiparas rather than primiparas. In this situation, the uterus must contract more forcefully to regain its prepregnancy size. These sensations are noticed most intensely while breastfeeding because the infant's sucking causes a release of oxytocin from the posterior pituitary, increasing the strength of the contractions. After the birth of the placenta, progesterone levels decrease which triggers milk production. Frequency of breastfeeding should not cause hyper-stimulation of the uterus and the woman may continue to feel contractions as long as she is breastfeeding, regardless of the presence or absence of blood clots.

A client has just expelled a hydatidiform mole. She's visibly upset over the loss and wants to know when she can try to become pregnant again. How should the nurse respond? "Why don't we discuss this with you at a later time when you're feeling better." "You must wait at least 1 year before becoming pregnant again." "Let me check with your physician and get you something that will help you relax." "Pregnancy should be avoided until all of your testing is normal."

"You must wait at least 1 year before becoming pregnant again." Explanation: Clients who develop a hydatidiform mole must be instructed to wait at least 1 year before attempting another pregnancy, despite testing that shows they have returned to normal. A hydatidiform mole is a precursor to cancer, so the client must be monitored carefully for 1 year by an experienced health care provider. Discussing this situation at a later time or checking with the physician to give the client something to relax does nothing to address the client's immediate concerns. Advising the client to wait until all tests are normal is a vague response and provides the client with little information.

The health care provider (HCP) plans to perform an amniotomy on a multiparous client admitted to the labor area at 41 weeks' gestation for labor induction. After the amniotomy, what should the nurse do first? Monitor the client's contraction pattern. Assess the fetal heart rate (FHR) for 1 full minute. Assess the client's temperature and pulse. Document the color of the amniotic fluid.

Assess the fetal heart rate (FHR) for 1 full minute. Explanation: After an amniotomy, the nurse should plan to first assess the FHR for 1 full minute. One of the complications of amniotomy is cord compression and/or prolapsed cord, and a FHR of 100 bpm or less should be promptly reported to the HCP. A cord prolapse requires prompt birth by cesarean section. The client's contraction pattern should be monitored once labor has been established. The client's temperature, pulse, and respirations should be assessed every 2 to 4 hours after rupture of the membranes to detect an infection. The nurse should document the color, quantity, and odor of the amniotic fluid, but this can be done after the FHR is assessed and a normal pattern is present.

A client with diabetes mellitus gives birth to a 9-lb, 10-oz (4,375 g) neonate at 38 weeks. What is the nurse's priority action after the stabilization of the neonate? Assess the neonate's reflexes. Provide glucose solution to neonate. Assess the neonate's weight. Assess the neonate's blood glucose level.

Assess the neonate's blood glucose level. Explanation: Glucose monitoring of the neonate born to a mother with diabetes is essential because the neonate is at risk for developing hypoglycemia after birth. The nurse should not provide glucose until after the blood glucose level is known to determine if it is needed. It is not necessary to immediately assess weight or reflexes.

The nurse is caring for a client in labor and notes late decelerations on the external fetal monitoring strip. Which actions will the nurse include in the client's plan of care? Select all that apply. Increase the oxytocin infusion. Contact the healthcare provider. Administer oxygen to the client. Discontinue the external fetal heart monitor. Change the client's position.

Contact the healthcare provider. Administer oxygen to the client. Change the client's position. Explanation: Late decelerations are caused by placental insufficiency and need to have an active response. Increasing the oxytocin would cause the fetus to have further placental insufficiency. Discontinuing the external fetal heart monitor would be inappropriate as the fetus needs intervention. Calling the healthcare provider would be necessary for a change in the plan of care. Application of oxygen to the client would help facilitate further oxygenation of the fetus. Changing the client's position may help alleviate the decelerations if they are caused by hypotension and will help determine if the decelerations are truly late.

A nurse is caring for a client who is 32 weeks gestation and being monitored in the antepartum unit for pre-eclampsia. The client suddenly reports continuous abdominal pain and vaginal bleeding. Which nursing interventions are priorities? Select all that apply. Evaluate maternal vital signs. Prepare for vaginal birth. Reassure the client that she will be able to continue the pregnancy. Auscultate fetal heart tones. Monitor the amount of vaginal bleeding. Monitor intake and output.

Evaluate maternal vital signs. Auscultate fetal heart tones. Monitor the amount of vaginal bleeding. Monitor intake and output. Explanation: The client's symptoms indicate that regular assessment is needed. The nurse must immediately evaluate the mother's well-being by evaluating vital signs; evaluate the well-being of the fetus by auscultating fetal heart tones; monitor the amount of blood loss; and evaluate volume status by monitoring intake and output. At this point, there is no indication of an imminent vaginal birth. It is important to offer comforting words to the client but not offer false reassurance.

A pregnant client arrives in the emergency department and states, "My baby is coming." The nurse sees a portion of the umbilical cord protruding from the vagina. What should the nurse do first? Assist the client to a stretcher. Hold pressure on the fetal head. Take the client to the labor and delivery unit. Notify the healthcare provider.

Hold pressure on the fetal head. Explanation: A prolapsed cord is an emergency situation. The most important function for the nurse is to hold pressure on the fetal head to decrease pressure on the cord and prevent further fetal compromise. While holding pressure, the nurse can call for help to assist the client to a stretcher and have someone notify the healthcare provider. Once this is done, the labor and delivery unit will be notified so that an operating room is ready for a cesarean delivery. The nurse should maintain pressure on the fetal head throughout the process to prevent occlusion of the umbilical cord.

What should the nurse include in the plan of care for a client with diabetes who is in labor? Measure urine output every 4 hours. Administer insulin subcutaneously every 4 hours. Check deep tendon reflexes every 2 hours. Monitor blood glucose levels every hour.

Monitor blood glucose levels every hour. Explanation: Because metabolic changes occur during labor and birth, close monitoring of the diabetic client's blood glucose level every hour during labor is necessary.There is no indication that the client's urine output needs to be measured every 4 hours.The need for insulin administration is determined by the client's blood glucose levels. If needed, insulin is usually given intravenously.Checking deep tendon reflexes would be necessary if the client had preeclampsia. However, because pregnant clients with diabetes are at higher risk for developing preeclampsia, any evidence of protein in the urine should be reported.

During the initial assessment, the nurse notes that the neonate's hands and feet appear blue while the neonate's torso appears pale pink. What should the nurse do next? Place the infant skin to skin with the mother. Ask the mother to massage the neonate's hands and feet. Keep the neonate in an isolation incubator for at least 2 hours. Report the neonate's cyanosis to the primary care provider promptly.

Place the infant skin to skin with the mother. Explanation: The neonate is demonstrating acrocyanosis, a normal finding evidenced by bluish hands and feet due to the neonate being cold or poor perfusion of the blood to the periphery of the body. The most appropriate action is to place the neonate skin to skin with the mother if stable, wrap the neonate in a warm blanket, or place the neonate under a radiant warmer. Massaging the extremities is inappropriate because it will not help to improve the circulation. Keeping the neonate in an isolation incubator is not warranted because acrocyanosis is not an infection but rather a manifestation of the neonate's sluggish peripheral circulation. Because acrocyanosis is a normal finding, notifying the primary care provider is not necessary.

The nurse is caring for a primigravida client who has been admitted to the labor and birth unit. Assessment reveals fetal malpresentation, green amniotic fluid, and a fetal heart rate (FHR) of 98 beats/minute. What is the nurse's priority intervention? Administer IV oxytocin as prescribed. Apply an internal fetal scalp electrode. Instruct the client to push. Prepare for an emergency cesarean birth.

Prepare for an emergency cesarean birth. Explanation: Because the abnormal FHR and amniotic fluid color suggest fetal distress, the nurse should prepare for an emergency cesarean birth. Giving oxytocin may increase fetal distress. Applying a fetal scalp electrode and having the client push would not address this emergency situation.

The nurse is caring for a multigravid client and observes the woman squatting on the bed and the fetal head crowning. After calling for assistance and helping the client lie down, the nurse should perform which action next? Tell the client to push between contractions. Provide gentle support to the fetal head. Apply gentle upward traction on the neonate's anterior shoulder. Massage the perineum to stretch the perineal tissues.

Provide gentle support to the fetal head. Explanation: During a precipitous birth, after calling for assistance and helping the client lie down, the nurse should provide support to the fetal head to prevent too rapid of emergence leading to injury. It is not appropriate to tell the client to push between contractions because this may lead to lacerations. The shoulder should be delivered by applying downward traction until the anterior shoulder appears fully at the introitus, then upward pressure to lift out the other shoulder. Priority should be given to safe birth of the infant over protecting the perineum by massage.

The physician orders docusate sodium 100 mg at bedtime for a primiparous client after vaginal delivery of a term neonate after a midline episiotomy. The nurse instructs the client to expect which of the following results from taking the medication? Relief from episiotomy pain. Contraction of the uterus. Softening of the stool. Aid in sleeping.

Softening of the stool. Explanation: Docusate sodium is a stool softener, used to assist in bowel elimination. The client is at risk for constipation because of decreased food and fluid intake and pain from the episiotomy. Numerous analgesics, such as ibuprofen or acetaminophen, could be used to treat episiotomy pain, helping the client achieve comfort and thus fall asleep. Oxytocin is used to contract the uterus.

A nurse is providing care for a pregnant client. The client asks the nurse how she can best deal with her fatigue. What instructions would the nurse give to this client? Take sleeping pills. Take an afternoon nap. Go to bed earlier at night. Sleep only on the left side.

Take a afternoon nap Fatigue is a common part of pregnancy. It is worst in the first and third trimesters; first-trimester fatigue is often associated with the many physical and psychosocial changes of being pregnant, and third-semester fatigue is caused by sleep disturbances from increased weight and physical discomforts such as heartburn. If physical causes of the fatigue are ruled out, the client should be arranging her life to permit additional rest periods. Naps should be encouraged. It is best to sleep on the left side, but a position of comfort improves sleep. Sleeping pills should be contraindicated during pregnancy. The client should go to bed to sleep on her natural sleep schedule. If one is used to going to bed late at night, then going to bed earlier doesn't mean the client will fall asleep earlier. The client should sleep and rest when her body tells her.

After an amniotomy, which client goal should take the highest priority? The client will express increased knowledge about amniotomy. The client will maintain adequate fetal tissue perfusion. The client will display no signs of infection. The client will report relief of pain.

The client will maintain adequate fetal tissue perfusion. Explanation: Amniotomy increases the risk of umbilical cord prolapse, which would impair the fetal blood supply and tissue perfusion. Because the fetus's life depends on the oxygen carried by that blood, maintaining fetal tissue perfusion takes priority over goals related to increased knowledge, infection prevention, and pain relief.

A client at 36 weeks' gestation, begins to exhibit signs of labor after an eclamptic seizure. The nurse should assess the client for: abruptio placentae transverse lie placenta accreta uterine atony

abruptio placentae Explanation: After an eclamptic seizure, the client is at risk for abruptio placentae due to severe vasoconstriction resulting in hemorrhage into the decidua basalis. Abruptio placentae is manifested by a board-like abdomen and an abnormal fetal heart rate tracing. Transverse lie or shoulder presentation, placenta accreta, and uterine atony are not related to eclampsia. Causes of a transverse lie may include relaxation of the abdominal wall secondary to grand multiparity, preterm fetus, placenta previa, abnormal uterus, contracted pelvis, and excessive amniotic fluid. Placenta accreta, a rare phenomenon, refers to a condition in which the placenta abnormally adheres to the uterine lining. Uterine atony, or relaxed uterus, may occur after childbirth, leading to postpartum hemorrhage.

A primigravid client has just completed a difficult, forceps-assisted birth of a 9-lb (4.08-Kg) neonate. Her labor was unusually long and required oxytocin augmentation. The nurse who's caring for her should stay alert for uterine inversion. atony. involution. discomfort.

atony. Explanation: A large fetus, extended labor, stimulation with oxytocin, and traumatic birth commonly are associated with uterine atony, which may lead to postpartum hemorrhage. Uterine inversion may precede or follow birth and commonly results from apparent excessive traction on the umbilical cord and attempts to deliver the placenta manually. Uterine involution and some uterine discomfort are normal after childbirth.

Which finding would the nurse most expect to find in a neonate born at 28 weeks' gestation who is diagnosed with intraventricular hemorrhage (IVH)? increased muscle tone hyperbilirubinemia bulging fontanels hyperactivity

bulging fontanels Explanation: A common finding of IVH is a bulging fontanel. The most common site of hemorrhage is the periventricular subependymal germinal matrix, where there is a rich blood supply and where the capillary walls are thin and fragile. Rapid volume expansion, hypercarbia, and hypoglycemia contribute to the development of IVH. Other common manifestations include neurologic signs such as hypotonia, lethargy, temperature instability, nystagmus, apnea, bradycardia, decreased hematocrit, and increasing hypoxia. Seizures also may occur. Hyperbilirubinemia refers to an increase in bilirubin in the blood and may be seen if bleeding was severe.

A client with gestational hypertension receives magnesium sulfate 50% 4 g in 250 mL D5W over 20 minutes. What priority assessment should the nurse perform when administering this drug? deep tendon reflexes temperature fetal heart rate intake and output

deep tendon reflexes Explanation: Magnesium sulfate is given to prevent and control seizures in clients with gestational hypertension. It is administered by IV; 4 g of a 50% solution in 250 mL D5W can be given as a bolus before the dose is titrated for continuous infusion. Magnesium sulfate is a general inhibitor of neurotransmission. As such, the two largest complications are the loss of deep tendon reflexes and the suppression of breathing. These are the priority assessments. If deep tendon reflexes decrease or the respiratory rate is 12 breaths/min or less, the medication should be discontinued and calcium gluconate administered. Magnesium sulfate is excreted entirely through the kidneys so intake and output should be evaluated hourly. The mother becomes very hot and flushed. This is a normal response. The fetal heart rate should not decrease from the drug.

A client at 32 weeks of gestation has mild preeclampsia. She is discharged home with instructions to remain in bed rest. She is also instructed to call her healthcare provider if she experiences which of the following symptoms? Select all that apply. headache increased urine output blurred vision difficulty sleeping epigastric pain severe nausea and vomiting

headache blurred vision epigastric pain severe nausea and vomiting Explanation: The care of a client with mild preeclampsia can be managed at home with proper instructions. Headache, blurred vision, epigastric pain, and severe nausea and vomiting can indicate worsening preeclampsia. Decreased, not increased, urine output is a concern because preeclampsia is associated with decreased renal perfusion, leading to a reduction in the glomerular filtration rate and decreased urine output. Difficulty sleeping, a common concern during the third trimester, is only a concern if it is caused by any of the other symptoms.

Which finding would lead the nurse to suspect that a neonate born at 34 weeks' gestation receiving intravenous fluids has developed overhydration? hypernatremia polycythemia hypoproteinemia increased urine specific gravity

hypoproteinemia Explanation: Decreased protein or hypoproteinemia is a sign of overhydration, which can lead to patent ductus arteriosus or congestive heart failure. Bulging fontanels, decreased serum sodium, decreased urine specific gravity, and decreased hematocrit are other signs of overhydration. Hypernatremia (increased serum sodium concentration) or increased urine specific gravity would suggest dehydration, not overhydration. Polycythemia evidenced by an elevated hematocrit would suggest hypoxia or congenital heart disorder.

A nurse is developing a care plan for a client in her 34th week of gestation who's experiencing premature labor. What nonpharmacologic intervention should the plan include to halt premature labor? encouraging ambulation serving a nutritious diet promoting adequate hydration performing nipple stimulation

promoting adequate hydration Explanation: Providing adequate hydration to the woman in premature labor may help halt contractions. The client should be placed on bed rest so that the fetus exerts less pressure on the cervix. A nutritious diet is important in pregnancy, but it won't halt premature labor. Nipple stimulation activates the release of oxytocin, which promotes uterine contractions.

A multiparous client, 28 hours after cesarean birth, who is breastfeeding has severe cramps or afterpains. The nurse explains that these are caused by which factor? flatulence accumulation after a cesarean birth healing of the abdominal incision after cesarean birth adverse effects of the medications administered after birth release of oxytocin during the breastfeeding session

release of oxytocin during the breastfeeding session Explanation: Breastfeeding stimulates oxytocin secretion, which causes the uterine muscles to contract. These contractions account for the discomfort associated with afterpains. Flatulence may occur after a cesarean birth. However, the mother typically would have abdominal distention and a bloating feeling, not a "cramping" feeling. Stretching of the tissues or healing may cause slight tenderness or itching, not cramping feelings of discomfort. Medications such as mild analgesics or stool softeners, commonly administered postpartum, typically do not cause cramping.

A neonate born by elective cesarean birth weighs 7 lb, 3 oz (3,267 g). The nurse closely monitors for which assessment finding related to a complication from this type of birth? fluctuating body temperature respiratory distress peripheral and circumoral cyanosis fluctuating blood glucose results

respiratory distress Explanation: One of the most crucial adaptations the newborn makes at birth is adjusting from a fluid-filled intrauterine environment to a gaseous extrauterine environment. The fluid from the intrauterine life environment must be removed so the infant can breathe. Passage through the birth canal allows intermittent compression of the thorax, which helps eliminate the fluid. If a mother has a cesarean birth, this compression does not occur, and transient tachypnea (respiratory rate above 60) can develop. Unstable blood sugars, poor thermoregulation, and peripheral and circumoral cyanosis (acrocyanosis) are associated with all newborns and not necessarily related to cesarean delivery.

The nurse is assessing a pregnant client with abruptio placenta. The nurse should notify the primary care provider about which finding? excessive vaginal bleeding rigid, board-like abdomen tetanic uterine contractions premature rupture of membranes

rigid, board-like abdomen Explanation: The most common assessment finding in a client with abruptio placenta is a rigid or board-like abdomen. Pain, usually reported as a sharp stabbing sensation high in the uterine fundus with the initial separation, also is common.Typically, bleeding is usually heavy, but it may not be visible or readily apparent depending on the area of separation.Tetanic uterine contractions are contractions that are excessively strong and last longer than 90 seconds. This type of contraction is usually associated with oxytocin toxicity.With abruptio placenta, the membranes usually do not rupture prematurely.

A primigravid client is admitted to the labor and delivery area, where the nurse evaluates her. Which assessment finding may indicate the need for cesarean birth? insufficient perineal stretching rapid, progressive labor umbilical cord prolapse fetal prematurity

umbilical cord prolapse Explanation: Indications for cesarean birth include umbilical cord prolapse, breech presentation, fetal distress, dystocia, previous cesarean birth, herpes simplex infection, condyloma acuminatum, placenta previa, abruptio placentae, and unsuccessful labor induction. Insufficient perineal stretching; rapid, progressive labor; and fetal prematurity aren't indications for cesarean birth.

A teen client, who is one week postpartum, is concerned about the possibility of postpartum depression because she has a history of depression. Which comment by the client would indicate that she understood the nurse's teaching about the postpartum period and her risks for postpartum depression? "Sleep shouldn't be too much of a problem, because the baby will soon start to sleep through the night." "Since I'm breastfeeding, I can eat all the food I want and not feel fat. The baby will use all the calories." "If I'm feeling guilty or not capable of caring for the baby and am not sleeping or eating well, I need to contact the office." "I'm going to give the baby the best care possible without asking anyone for help to show all those people who think I can't do it."

"If I'm feeling guilty or not capable of caring for the baby and am not sleeping or eating well, I need to contact the office." Explanation: Feelings of guilt combined with a lack of self-care (not eating or sleeping enough) can predispose a new mother to postpartum depression, especially one who has had previous episodes of depression. Sleep is essential to both the mother and baby, but sleeping through the night does not usually occur in the first few weeks after birth. While breastfeeding mothers do need good nutrition, unlimited eating after childbirth may inhibit the return to a normal weight and could create depression in a new mother, especially a vulnerable one. Attempting to care for an infant with no help from others is likely to cause stress that could lead to depression, especially in an adolescent.

A 29-year-old multigravida at 37 weeks' gestation is being treated for severe preeclampsia and has magnesium sulfate infusing at 3 g/h. What is the priority intervention to maintain safety for this client? Maintain continuous fetal monitoring. Encourage family members to remain at bedside. Assess reflexes, clonus, visual disturbances, and headache. Monitor maternal liver studies every 4 hours.

Assess reflexes, clonus, visual disturbances, and headache. Explanation: The central nervous system (CNS) functioning and freedom from injury is a priority in maintaining well-being of the maternal-fetal unit. If the mother suffers CNS damage related to hypertension or stroke, oxygenation status is compromised, and the well-being of both mother and infant are at risk. Continuous fetal monitoring is an assessment strategy for the infant only and would be of secondary importance to maternal CNS assessment because maternal oxygenation will dictate fetal oxygenation and well-being. In preeclampsia, frequent assessment of maternal reflexes, clonus, visual disturbances, and headache give clear evidence of the condition of the maternal CNS system. Monitoring the liver studies does give an indication of the status of the maternal system, but the less invasive and highly correlated condition of the maternal CNS system in assessing reflexes, maternal headache, visual disturbances, and clonus is the highest priority. Psychosocial care is a priority and can be accomplished in ways other than having the family remain at the bedside.

For a primigravid client with the fetal presenting part at -1 station, what would be the nurse's priority immediately after a spontaneous rupture of the membranes? Position the client on her left side. Prepare the client for a cesarean birth. Assess the client's blood pressure. Check the fetal heart rate.

Check the fetal heart rate. Explanation: Immediately after a spontaneous rupture of the membranes, the nurse should listen to the fetal heart rate to detect bradycardia. With the fetus at -1 station, the cord may prolapse as amniotic fluid rushes out. Fetal heart rate should be monitored because it will indicate if cord prolapse or cord compression has occurred. The color, amount, and odor of the amniotic fluid should be noted.Although the optimal position for the client is side lying, this is not a priority at this time.The client is not having a precipitous birth with the fetal head at ?1 station. Therefore, preparing the client for a cesarean birth is unnecessary.Although maternal blood pressure should be monitored throughout labor, this is not a priority at this time.

The primary care provider prescribes a tocolytic for a pregnant client with premature rupture of the membranes who begins to have contractions every 10 minutes. The drug has had expected effects when the nurse observes which finding? The client is sedated. There is increased placental perfusion. There is improvement in fetal lung function. Contractions cease.

Contractions cease. Explanation: Tocolytics are used to stop uterine contractions. Sedation is not its purpose of a tocolytics. Tocolytics have no effect on placental perfusion or the fetal pulmonary system or lung function.

The nurse is reviewing a client's prenatal history. Which of the following is a significant factor in anticipating complications in labor and birth? History of postpartum hemorrhage (PPH) Urinary tract infection at 16 weeks gestational age Gravida 4, Para 3 Amniocentesis performed at 14 weeks gestational age

History of postpartum hemorrhage (PPH) Explanation: Women who have a history of PPH are at higher risk for a PPH in subsequent pregnancies. This is a significant factor for the nurse to know in planning and being prepared for the birth of the baby because this is the client's fourth labor and birth. Urinary tract infections may occur during pregnancy as the enlarging uterus puts pressure on the ureters, resulting in urinary stasis. However, there is not a significant impact on labor and birth. Following amniocentesis, the client may experience cramping and feelings of increased pressure. The associated complications with amniocentesis are proximal (infection, fetal injury, bleeding) but do not pose a long-term consequence in relation to the labor process.

While performing a cervical examination on a client in labor, a nurse's fingertips feel pulsating tissue. What is the most appropriate nursing intervention? Leave the client and call the provider. Put the client in a semi-Fowler's position. Ask the client to push with the next contraction. Leave the fingers in place and press the nurse call light.

Leave the fingers in place and press the nurse call light. Explanation: When the umbilical cord precedes the fetal presenting part, it's known as a prolapsed cord. Leaving the fingers in place and calling for assistance is the safest intervention for the fetus. The nurse will need to keep the fetus off the cord to reduce cord compression. The nursing staff will contact the provider, and the client will probably require a cesarean birth to decrease the risk of fetal demise during birth. Placing the client in the semi-Fowler's position would increase the pressure of the fetus on the umbilical cord. Asking the client to push with the next contraction would force the presenting part against the cord, causing severe bradycardia and possible fetal demise.

A client at 30 weeks gestation experiences a rupture of membranes with mild contractions 8 minutes apart. Which nursing interventions are included on the plan of care to improve newborn outcomes? Select all that apply. Maintain the client on the fetal monitor throughout the labor process. Administer a dose of betamethasone per healthcare provider's order. Arrange a neonatologist to be available for the birth. Ensure the mother remains nothing by mouth (NPO) throughout the labor. Position the mother in a supine position with the feet elevated. Begin an oxytocin drip once the mother reaches 3 cm dilated.

Maintain the client on the fetal monitor throughout the labor process. Administer a dose of betamethasone per healthcare provider's order. Arrange a neonatologist to be available for the birth. Explanation: The nurse caring for a client at 30 weeks gestation who has a rupture of membranes realizes that preparation is needed for a premature delivery. To improve newborn outcomes, the nurse must be aware of the status of the fetus via a fetal monitor, administer betamethasone (it is best to have at least two doses 12 hours apart) to increase the surfactant level and fetal lung maturity, and have resuscitation equipment available, if needed. It is best have a neonatologist present as well to assess the neonate and plan medical care. The mother typically is permitted to have ice chips at most in case emergency surgery is needed. The mother is not placed in a supine position as there is the potential of compressing the vena cava causing maternal hypotension and reduced blood flow to the fetus. Oxytocin is naturally produced by the posterior pituitary with Pitocin being the synthetic version. It is used to stimulate contractions. Stimulating contractions is not indicated at this time.

The nurse is monitoring a client receiving oxytocin. What is one possible scenario in which this drug would be indicated and used? Medically indicated induction of labor in client at 39 weeks' gestation Relief of hyperemesis gravidarum in a client at 10 weeks' gestation Treatment of dysfunctional uterine bleeding in a client who is not pregnant Cessation of premature labor in the client at 24 weeks' gestation

Medically indicated induction of labor in client at 39 weeks' gestation Explanation: Oxytocin is a medication that is used in the induction and augmentation of labor, stimulating uterine contractions. It is also used to reduce bleeding after childbirth. Oxytocin would be appropriate in the medically indicated induction for the client who is at 39 weeks' gestation, as it would help induce labor and stimulate uterine contractions. Antiemetics and intravenous fluids would be used to relieve symptoms of hyperemesis gravidarum. Treatment of dysfunctional uterine bleeding in the nonpregnant client would include hormone therapy such as oral contraceptives. Oxytocin would not halt preterm labor; it would augment labor and result in the preterm birth of the fetus.

Umbilical cord prolapse occurs after spontaneous rupture of the membranes. What should the nurse do immediately? Place the client in a Trendelenburg position. Administer oxytocin intravenously. Ask the client to begin pushing. Cover the cord with sterile towels.

Place the client in a Trendelenburg position. Explanation: The first step in managing a cord prolapse is to relieve pressure on the cord. Immediate measures include lowering the client's head by using the Trendelenburg position or knee-to-chest position so that the fetal presenting part will move away from the pelvis and moving the fetal presenting part off the cord by applying pressure through the vagina with a sterile gloved hand. An immediate cesarean birth is usually performed.Oxytocin would not be given because the drug stimulates uterine contractions, putting further pressure on the cord as the contractions attempt to expel the fetus.Pushing results in further cord compression and decreased fetal heart rate.With cord prolapse, an immediate cesarean birth is indicated. There is no need to cover the cord to avoid damage or tearing.

A 24-year-old primigravid client who gives birth to a viable term neonate is prescribed to receive oxytocin intravenously after delivery of the placenta. Which of the following signs would indicate to the nurse that the placenta is about to be delivered? The cord lengthens outside the vagina. There is decreased vaginal bleeding. The uterus cannot be palpated. The uterus changes to discoid shape.

The cord lengthens outside the vagina. Explanation: The most reliable sign that the placenta has detached from the uterine wall is lengthening of the cord outside the vagina. Other signs include a sudden gush of (rather than a decrease in) vaginal blood. Usually, when placenta detachment occurs, the uterus becomes firmer and changes in shape from discoid to globular. This process takes about 5 minutes. If the placenta does not separate, manual removal may be necessary to prevent postpartum hemorrhage.

The nurse is admitting a primigravid client at 37 weeks' gestation who has been diagnosed with preeclampsia to the labor and birth area. Which client care rooms is most appropriate for this client? a brightly lit private room at the end of the hall from the nurses' station a semiprivate room midway down the hall from the nurses' station a private room with many windows that is near the operating room a darkened private room as close to the nurses' station as possible

a darkened private room as close to the nurses' station as possible Explanation: A primigravid client diagnosed with preeclampsia has the potential for developing seizures (eclampsia). This client should be in a room with the least amount of stimulation possible to reduce the risk of seizures and as close to the nurses' station as possible in case the client requires immediate assistance. Bright lighting and sunshine can be a stimulant, possibly increasing the risk of seizures, as can being in a semiprivate room with roommate, visitors, conversation, and noise.

Early detection of an ectopic pregnancy is paramount in preventing a life-threatening rupture. Which symptoms should alert the nurse to the possibility of an ectopic pregnancy? abdominal pain, vaginal bleeding, and a positive pregnancy test nausea and vomiting amenorrhea and a negative pregnancy test copious discharge of clear mucous and prolonged epigastric pain

abdominal pain, vaginal bleeding, and a positive pregnancy test Explanation: Abdominal pain, vaginal bleeding, and a positive pregnancy test are cardinal signs of an ectopic pregnancy. Nausea and vomiting may occur before rupture, but significantly increase after rupture. Amenorrhea and a negative pregnancy test may indicate a metabolic disorder such as hypothyroidism. Discharge of clear mucous isn't indicative of an ectopic pregnancy and referred shoulder pain, not epigastric pain, is expected.

Which outcome would the nurse identify as the priority to achieve when developing the plan of care for a primigravid client at 38 weeks' gestation who is hospitalized with severe preeclampsia and receiving intravenous magnesium sulfate? decreased generalized edema within 8 hours decreased urinary output during the first 24 hours sedation and decreased reflex excitability within 48 hours absence of any seizure activity during the first 48 hours

absence of any seizure activity during the first 48 hours Explanation: The highest priority for a client with severe preeclampsia is to prevent seizures, thereby minimizing the possibility of adverse effects on the mother and fetus, and then to facilitate safe childbirth. Efforts to decrease edema, reduce blood pressure, increase urine output, limit kidney damage, and maintain sedation are desirable but are not as important as preventing seizures. It would take several days or weeks for the edema to be decreased. Sedation and decreased reflex excitability can occur with the administration of intravenous magnesium sulfate, which peaks in 30 minutes, much sooner than 48 hours.

At 6 cm dilation, a client in labor receives a lumbar epidural for pain control. Which nursing diagnosis is most appropriate? risk for injury related to rapid delivery acute pain related to wearing off of anesthesia hyperthermia related to effects of anesthesia altered tissue perfusion related to effects of anesthesia

altered tissue perfusion related to effects of anesthesia Explanation: A disadvantage of lumbar epidural is the risk for hypotension, which can lead to altered tissue perfusion. Epidurals are associated with longer labor and hypothermia, not rapid delivery and hyperthermia. Any pain the client experiences wouldn't be directly related to the wearing off of anesthesia.

The nurse assists with a precipitous birth in an outpatient setting. While waiting for more advanced care, the nurse places the infant skin to skin with the mother and encourages breastfeeding. What are the desired outcomes of skin-to-skin care with early breastfeeding? Select all that apply. beginning the parental-infant bonding process preventing neonatal hypothermia providing glucose to the neonate contracting the mother's uterus preventing maternal infection

beginning the parental-infant bonding process preventing neonatal hypothermia providing glucose to the neonate contracting the mother's uterus Explanation: The nurse places the newborn skin to skin with the mother immediately following birth for many reasons. The practice facilitates transition to the extra uterine environment. Skin-to-skin contact helps to begin the parental-infant bonding process, and helps prevent neonatal hypothermia. Early breastfeeding provides the neonate with nutrients to prevent hypoglycemia. Breastfeeding stimulates the natural production of oxytocin, which helps reduce the potential for uterine atony. Skin-to-skin care with early breastfeeding does not reduce the risk of maternal infections.

The nurse is caring for a client prescribed a tocolytic agent. The nurse takes immediate action based on what assessment finding? blood glucose of 170 mg/dL (9.4 mmol/L) maternal heart rate of 114 beats/min bilateral crackles on lung auscultation peripheral pulse strength of +2

bilateral crackles on lung auscultation Explanation: Tocolytics are used to stop labor contractions. The most common adverse effect associated with the use of these drugs is pulmonary edema. Bilateral crackles on lung auscultation is a sign of pulmonary edema, and prompt action would be required. A serum glucose level of 170 mg/dL (9.4 mmol/L) is elevated and should be reported, but it is not life-threatening. Tocolytics may cause tachycardia and increased cardiac output with bounding arterial pulsations. A peripheral pulse strength of +2 indicates a slightly lower than normal level that is not an immediate cause for concern.

A client has given birth to a preterm neonate. The client tells the nurse that she wants to breast-feed her neonate. The nurse should explain to the mother that breast milk contains antibodies that help protect her neonate. commercial formula will provide better nutrition for the neonate. breast-feeding can be started when the neonate is ready for discharge. the neonate will be less likely to develop an infection on commercial formula.

breast milk contains antibodies that help protect her neonate. Explanation: Studies have proven that breast milk provides preterm neonates with better protection from infections such as necrotizing enterocolitis because of the antibodies contained in the milk. Commercial formula doesn't provide any better nutrition than breast milk. Breast milk feedings can be started as soon as the neonate is stable. The neonate is more likely to develop infections when fed formula rather than breast milk.

hyperemesis gravidarum Explanation: Hyperemesis gravidarum differs from the nausea and vomiting (morning sickness) that normally occur during pregnancy. It's characterized by excessive vomiting that can lead to dehydration and starvation. Without treatment, metabolic changes can lead to severe complications, even death, of the fetus or mother. Eclampsia is the most serious form of gestational hypertension. It's characterized by hypertension, seizures, coma, edema, and proteinuria. Hydramnios is an overproduction of amniotic fluid that causes uterine distension.

calcium gluconate Explanation: The nurse should anticipate administering 10 ml of 10% calcium gluconate by I.V. push over 3 to 5 minutes as a calcium gluconate antidote for magnesium toxicity. Hydralazine/hydralazine is given for sustained elevated blood pressures in clients with preeclampsia. Naloxone is used to correct opioid toxicity. Rohm(D) immune globulin is given to clients with Rh-negative blood to prevent antibody formation from Rh-positive fetuses.

A client who is 15 weeks pregnant comes to the clinic for amniocentesis. The nurse knows that this test can be used to identify which characteristics or problems? Select all that apply. fetal lung maturity gestational diabetes chromosomal defects neural tube defects polyhydramnios sex of the fetus

chromosomal defects neural tube defects sex of the fetus Explanation: In early pregnancy, amniocentesis can be used to identify chromosomal and neural tube defects and to determine the sex of the fetus. It can also be used to evaluate fetal lung maturity during the last trimester of pregnancy. According to the US Preventive Health Task Force, a blood test performed after 24 weeks of gestation is used to screen for gestational diabetes. Ultrasound is used to identify polyhydramnios; amniocentesis can be used to treat polyhydramnios by removing excess fluid

When caring for the neonate weighing 4,564 g (10 lb, 1 oz) born vaginally to a woman with diabetes, the nurse should assess the neonate for fracture of which area? clavicle skull wrist rib cage

clavicle Explanation: Infants born to mothers with diabetes tend to be larger than average, and this neonate weighs 10 lb, 1 oz (4,564 g). The most common fractures are those of the clavicle and long bones, such as the femur. In a neonate, the skull bones are not fused and move to allow for vaginal birth, so skull fracture is rarely seen. Wrist and rib cages are rarely fractured.

For the client who is receiving intravenous magnesium sulfate for severe preeclampsia, which assessment findings would alert the nurse to suspect hypermagnesemia? decreased deep tendon reflexes cool skin temperature rapid pulse rate tingling in the toes

decreased deep tendon reflexes Explanation: Typical signs of hypermagnesemia include decreased deep tendon reflexes, sweating or a flushing of the skin, oliguria, decreased respirations, and lethargy progressing to coma as the toxicity increases. The nurse should check the client's patellar, biceps, and radial reflexes regularly during magnesium sulfate therapy. Cool skin temperature may result from peripheral vasodilation, but the opposite—flushing and sweating—are usually seen. A rapid pulse rate commonly occurs in hypomagnesemia. Tingling in the toes may suggest hypocalcemia, not hypermagnesemia.

When assessing a neonate 1 hour after birth, the nurse notes acrocyanosis of both feet and hands, measures an axillary temperature of 95.5°F (35.3°C), an apical pulse of 110 beats/minute, and a respiratory rate of 64 breaths/minute. Which assessment would be the most concerning for the nurse? hypothermia tachycardia bradypnea hypoxia

hypothermia Explanation: The neonate's normal axillary temperature should range from approximately 97.7°F to 99.5°F (36.5°C to 37.5°C). A temperature of 95.5°F (35.3°C) is very low. When the temperature drops, the neonate is at risk for hypothermia, respiratory distress, and hypoglycemia. The normal respiratory rate for a newborn is 30 to 60 breaths/minute while resting. It can increase with crying, and it will increase if hypothermia develops. This neonate would have tachypnea instead of bradypnea. The normal heart rate for a newborn is 110 to 160 beats/minute, so 110 beats/minute would be a normal finding and not tachycardia. All neonates have acrocyanosis of the hands and feet in the first few hours of life; this would not indicate hypoxia.

The health care provider (HCP) orders an amniocentesis for a primigravid client at 37 weeks' gestation to determine fetal lung maturity. Which is an indicator of fetal lung maturity? amount of bilirubin present presence of red blood cells Barr body determination lecithin-sphingomyelin (L/S ratio)

lecithin-sphingomyelin (L/S ratio) Explanation: To determine fetal lung maturity, the sample of amniotic fluid will be tested for the L/S ratio. When fetal lungs are mature, the ratio should be 2:1. Bilirubin indicates hemolysis and, if present in the fluid, suggests Rh disease. Red blood cells should not appear in the amniotic fluid because their presence suggests fetal bleeding. Barr body determination is a chromosome analysis of the sex chromosomes that is sometimes used when a child is born with ambiguous genitalia.

The membranes of a multigravid client in active labor rupture spontaneously, revealing greenish-colored amniotic fluid. How does the nurse interpret this finding? passage of meconium by the fetus maternal intrauterine infection Rh incompatibility between mother and fetus maternal sexually transmitted disease

passage of meconium by the fetus Explanation: Greenish-colored amniotic fluid is caused by the passage of meconium, usually secondary to a fetal insult during labor. Meconium passage also may be related to an intact gastrointestinal system of the neonate, especially those neonates who are full term or of postdate gestational age. Amnioinfusion may be used to treat the condition and dilute the fluid. Cloudy amniotic fluid is associated with an infection caused by bacteria or a sexually transmitted disease. Severe yellow-colored fluid is associated with Rh incompatibility or erythroblastosis fetalis.

A 16-year-old unmarried client visiting the prenatal clinic at 32 weeks' gestation and currently weighing 140 lb (63.5 kg) is being closely monitored for early signs of preeclampsia. The client is 5 feet, 2 inch (158 cm) tall and weighed 120 lb (54.4 kg) before the pregnancy. Which factor would be most important to assess? proteinuria small-for-gestational-age fetus ABO incompatibility fluid intake

proteinuria Explanation: Because the client is being closely monitored for early signs of preeclampsia, checking the urine for proteinuria is most important. Proteinuria, even in the absence of an elevated blood pressure, is indicative of preeclampsia.Although adolescent pregnancy is associated with an increase in the number of small-for-gestational-age fetuses, this is not indicative of preeclampsia.ABO incompatibility, occurring when the mother has type O blood and the fetus is type A, B, or AB blood, is not associated with preeclampsia. Fluid intake is an important assessment for any pregnant client.However, it is not a primary indication of preeclampsia. Edema of the hands and face is a more important indicator than fluid intake.

A nurse is caring for a newborn exposed to drugs while in utero. Which behaviors will the nurse expect the newborn to exhibit? Select all that apply. tachypnea with excessive secretions effective latch to the breast easily consoled and comforted sensitive gag reflex hyperactivity and increased muscle tone

tachypnea with excessive secretions sensitive gag reflex hyperactivity and increased muscle tone Explanation: Newborns exposed to drugs while in utero can have tachypnea, excessive secretions, a sensitive gag reflex, hyperactivity, and increased muscle tone. Newborns exposed to drugs while in utero will not be satisfied with breastfeeding or eating and are not easily consoled or comforted.

A nurse obtains the antepartum history of a client who is 6 weeks pregnant. Which finding is a concern? the client's participation in low-impact aerobics three times per week the client's consumption of six to eight cans of beer on weekends the client's consumption of four to six small meals daily the client's practice of taking a multivitamin supplement daily

the client's consumption of six to eight cans of beer on weekends Explanation: Consuming any amount or type of alcohol isn't recommended during pregnancy because it increases the risk of fetal alcohol syndrome or fetal alcohol effect. The nurse should teach the client about these risks. If the client is accustomed to moderate exercise, she may continue to engage in low-impact aerobics during pregnancy. Eating frequent, small meals helps maintain the client's energy level by keeping the blood glucose level relatively constant. Taking a multivitamin supplement daily and eating a balanced diet are recommended during pregnancy.

A client undergoes an amniotomy. Shortly afterward, the nurse detects large variable decelerations in the fetal heart rate (FHR) on the external electronic fetal monitor (EFM). These findings may signify: infection. umbilical cord prolapse. start of the second stage of labor. need for labor induction.

umbilical cord prolapse. Explanation: After an amniotomy, a significant change in the FHR, particularly large variable decelerations associated with cord compression, may indicate umbilical cord prolapse. This movement of the umbilical cord relative to the fetal presenting part is an obstetric emergency that requires immediate intervention to prevent fetal hypoxia. Infection, the start of the second stage of labor, and the need for labor induction aren't associated with FHR changes. An infection causes temperature elevation. The second stage of labor starts with complete cervical dilation. Labor induction is indicated if the client's labor fails to progress.

Discharge planning is being finalized for a neonate who was born at 32 weeks' gestation and was diagnosed with retinopathy of prematurity. What should the nurse tell the parents? "An ophthalmologist will examine the baby before discharge." "You should schedule an appointment with a optometrist when the baby is 6 months old. "Before your child enters the public school system, you must arrange for an individualized educational plan with the school nurse." "Contact the local support group for the blind."

"An ophthalmologist will examine the baby before discharge." Explanation: An ophthalmologist commonly examines neonates with retinopathy of prematurity before discharge. Serial eye examinations are then necessary to determine the extent of damage. An optometrist can't provide follow-up treatment for the neonate with retinopathy of prematurity because some neonates require cryotherapy and laser photocoagulation therapy, both of which must be performed by an ophthalmologist. The parents should contact the early intervention program to set up an individualized educational plan for their child before he reaches school age. Because the neonate may have permanent vision loss, intervention before school age is important to the child's growth and development. The school nurse is only involved with individualized educational plans for children of school age. The neonate may not be blind, so suggesting a support group for the blind is inappropriate.

A client in active labor asks the nurse why her blood pressure is being monitored so frequently. What is the most appropriate response by the nurse? "It is part of our standard policy." "Changes in your blood pressure can affect the fetus." "Low blood pressure may cause dizziness and fainting." "Increased blood pressure is a sign of preeclampsia."

"Changes in your blood pressure can affect the fetus." Explanation: During contractions, blood pressure increases and blood flow to the intervillous spaces decreases, compromising the fetal blood supply. Therefore, the nurse should frequently assess the client's blood pressure to determine whether it returns to precontraction levels and allows adequate fetal blood flow. Preeclampsia causes the blood pressure to increase, and low blood pressure may cause dizziness; however, neither fact explains the primary reason for frequent monitoring. Telling the client that it is policy is not a patient-centered response.

A 40-year-old client at 8 weeks' gestation has a 3-year-old child with Down syndrome. The nurse is discussing amniocentesis and chorionic villus sampling as genetic screening methods for the expected baby. The nurse is confident that the teaching has been understood when the client makes which statement? "Each test identifies a different part of the infant's genetic makeup." "Chorionic villus sampling can be performed earlier in pregnancy." "The test results take the same length of time to be completed." "Amniocentesis is a more dangerous procedure for the fetus."

"Chorionic villus sampling can be performed earlier in pregnancy." Explanation: Chorionic villus sampling (CVS) can be performed from approximately 8 to 12 weeks' gestation, while amniocentesis cannot be performed until between 11 weeks' gestation and the end of the pregnancy. Eleven weeks' gestation is the earliest possible time within the pregnancy to obtain a sufficient amount of amniotic fluid to sample. Because CVS takes a piece of membrane surrounding the infant, this procedure can be completed earlier in the pregnancy. Amniocentesis and chorionic villus sampling identify the genetic makeup of the fetus in its entirety, rather than a portion of it. Laboratory analysis of chorionic villus sampling takes less time to complete. Both procedures place the fetus at risk and postprocedure teaching asks the client to report the same complicating events (bleeding, cramping, fever, and fluid leakage from the vagina).

A nurse is caring for a newborn with transient tachypnea of the newborn (TTN). Which responses made by the newborn's mother demonstrates that she understands the newborn's condition? Select all that apply. "Having a cesarean section increased the risk of transient tachypnea of my newborn." "My newborn is on oxygen because I was exposed to smoke during pregnancy." "I will need to give supplemental oxygen to my baby for the next 6 months." "The healthcare provider will need chest X-rays to monitor my baby's respiratory distress." "I can feed my newborn even though the respiratory rate is fast."

"Having a cesarean section increased the risk of transient tachypnea of my newborn." "The healthcare provider will need chest X-rays to monitor my baby's respiratory distress." Explanation: TTN is caused by retention of extra amniotic fluid in the lung fields. Newborns who are born by cesarean section are more likely to have TTN because the mechanical squeezing of the newborn's rib cage does not occur with birth. The most distinguishing feature of TTN is the lung fields on a chest X-ray where there is hyperaeration of the alveoli. TTN usually lasts 48-72 hours and newborns will need oxygen during this time period. TTN usually requires a low percentage of oxygenation (40% or less) along with supportive care such as IV fluid, antibiotics, and placement in a warmer. If the respiratory rate is consistently above 60 breaths/minute, the newborn takes nothing by mouth due to the risk of aspiration. Although smoking during pregnancy can lead to a smaller baby, smoking is not linked with TTN.

The nurse is caring for a client in the first stage of labor who states, "I am scared. The last time I gave birth, I was in labor for 32 hours, it was awful." What is the nurse's best response? "Let me call the anesthesiologist and get you an epidural right away." "Let's talk about options that can keep you more comfortable this time." "Your second labor should be much shorter." "Maybe we can start some oxytocin to speed things up."

"Let's talk about options that can keep you more comfortable this time." Explanation: A client's ability to cope during labor and childbirth may be hampered by fear of a long, painful or difficult childbirth. A previous negative experience may increase these fears. The client's anxiety stems from her past history of a long labor. Engaging the client in planning options will give more control and let her know that the nurse will support her through this labor. An immediate epidural may lengthen labor time, the nurse cannot know that labor will be shorter. Oxytocin is not an appropriate option at this time.

A client two days postpartum was given a shot of RhoGAM. At the postpartum home visit, the client asks the nurse why she needed RhoGAM. What is the most appropriate response by the nurse? "RhoGAM suppresses antibody formation in women with Rh positive blood after giving birth to an Rh negative baby." "RhoGAM suppresses antibody formation in women with Rh negative blood after giving birth to an Rh positive baby." "RhoGAM suppresses antibody formation in women with Rh positive blood after giving birth to an Rh positive baby." "RhoGAM suppresses antibody formation in women with RH negative blood after giving birth to an Rh negative baby."

"RhoGAM suppresses antibody formation in women with Rh negative blood after giving birth to an Rh positive baby." Explanation: RhoGAM is indicated to suppress antibody formation in women with Rh negative blood after giving birth to an RH positive baby. It is also given to Rh negative women after miscarriage/pregnancy termination, abdominal trauma, ectopic pregnancy, and amniocentesis.

A nurse is teaching a young adult female client about self-management of systemic lupus erythematosus (SLE). The client is prescribed ibuprofen, hydroxychloroquine, and cyclophosphamide. The client asks the nurse about the possibility of becoming pregnant while being treated for SLE. What is the nurse's best response? "You should speak with your healthcare provider about alternatives to taking cyclophosphamide." "You need to speak to the healthcare provider about stopping all medications prior to trying to become pregnant." "Unfortunately, having SLE greatly increases your risk for miscarriage and infertility." "Some women successfully have children, but the risk of passing on SLE is quite high."

"You should speak with your healthcare provider about alternatives to taking cyclophosphamide." Explanation: The most important piece of information in this moment is the advice to prevent pregnancy while on cyclophosphamide and to encourage the client to explore safer options if she wishes to become pregnant in the future. This drug is pregnancy category D, and there is a high risk of long-term fertility issues for women who take this medication. Tell the client that there may be a more appropriate alternative. Antimalaria medications such as hydroxychloroquine are usually continued during pregnancy, and ibuprofen poses a small risk during pregnancy. While there is a higher risk for miscarriage and fertility issues, this is not priority information for the client in this moment. While there is some genetic component to how SLE develops, there is no clear or strong pattern of heredity.

When teaching a primigravid client about the diagnostic tests used in pregnancy, the nurse should include which information? A fetal biophysical profile involves assessments of breathing movements, body movements, tone, amniotic fluid volume, and fetal heart rate reactivity. Fetal heart rate increases during a nonstress test is an ominous sign and requires further evaluation with fetal echocardiography. Contraction stress testing, performed on most pregnant women, can be initiated as early as 16 weeks' gestation. Percutaneous umbilical blood sampling uses a needle inserted through the vagina to obtain a sample.

A fetal biophysical profile involves assessments of breathing movements, body movements, tone, amniotic fluid volume, and fetal heart rate reactivity. Explanation: The fetal biophysical profile includes fetal breathing movements, fetal body movements, tone, amniotic fluid volume, and fetal heart rate reactivity. Normal nonstress test findings include at least two qualifying accelerations in the fetal heart rate from baseline in 20 minutes. A contraction stress test or oxytocin challenge test should be performed only on women who are at risk for fetal distress during labor. The contraction stress test is rarely performed before 28 weeks' gestation because of the possibility of initiating labor. Percutaneous umbilical cord sampling requires the insertion of a needle through the abdomen to obtain a fetal blood sample.

A multigravid client is receiving oxytocin augmentation. When the client's cervix is dilated to 6 cm, her membranes rupture spontaneously with meconium-stained amniotic fluid. Which action should the nurse perform first? Increase the rate of the oxytocin infusion. Turn the client to a knee-to-chest position. Assess cervical dilation and effacement. Assess the fetal heart rate.

Assess the fetal heart rate. Explanation: Assessing the fetal heart rate is always a priority after spontaneous rupture of membranes has occurred. Meconium-stained fluid is also a common sign of fetal distress related to an inadequate transfer of oxygen to the fetus. Because the fetus has suffered hypoxia, close fetal heart rate monitoring is necessary. In addition, all clients are monitored continuously after rupture of membranes for fetal distress caused by cord prolapse. If there are increasing signs of fetal distress (e.g., late decelerations), the health care provider (HCP) should be notified immediately. A cesarean birth may be performed for fetal distress. Increasing the rate of the oxytocin infusion could lead to further fetal distress. Turning the client to the left side, rather than a knee-chest position, improves placental perfusion. The HCP may wish to determine the extent of cervical dilation to make a decision about whether a cesarean birth is warranted, but continuous fetal heart rate monitoring is essential to determine fetal status.

The nurse is assessing a 2-hour-old newborn. The nurse notes nasal flaring and acrocyanosis. Which of the following is the nurse's priority intervention? Obtain blood pressure measurements on all four of the newborn's extremities. Assess the newborn's oxygen saturation. Palpate the newborn's fontanels. Auscultate the newborn's heart tones.

Assess the newborn's oxygen saturation. Explanation: Nasal flaring is a sign of respiratory distress in the newborn. Acrocyanosis is a normal finding in the newborn during transition. Assessment of the newborn's oxygen saturation is the only intervention that will assess the degree of respiratory distress. Blood pressure would be a very late sign of respiratory distress. Heart tones would not change nor would the fontanels.

The nurse is assessing a postpartum client who gave birth 6 hours earlier. The nurse notes the fundus to be deviated to the left side of the umbilicus. What action would the nurse take next? Request an order to increase the client's oxytocin infusion. Encourage the client to take a brief nap, and reassess the fundus in 1 hour. Assist the client to the bathroom to void, and reassess the fundus. Assess the perineum for swelling and bruising.

Assist the client to the bathroom to void, and reassess the fundus. Explanation: Elevation of the fundus and deviation to the side are evidence that the client's bladder is full. Having the client void and reassessing the fundus would be the appropriate intervention. Increasing the oxytocin will cause the fundus to contract but not decrease the fundal level if the bladder continues to be full. Reassessment of the fundus in 1 hour does not address the cause. Assessing the perineum checks for bleeding, but it does not address the cause.

A client is admitted to the labor area for induction with intravenous oxytocin because she is 42 weeks pregnant. What should the nurse include in the induction teaching plan for this client? Continuous fetal heart rate monitoring will be implemented. Frequent ultrasound examinations will be performed. At least 5 to 10 fetal scalp pH tests will be performed. Oligohydramnios will be carefully evaluated.

Continuous fetal heart rate monitoring will be implemented. Explanation: Uteroplacental insufficiency is associated with a postterm fetus; therefore, it is recommended that the fetal heart rate and contraction pattern be monitored throughout the labor and birth process. In addition, intravenous oxytocin, which is frequently used for induction of labor, may result in hyperstimulation of the uterus. Therefore, monitoring the client is critical.One ultrasound may be performed to assess position and confirm gestational age.A scalp pH may be performed if there is evidence of fetal bradycardia, late decelerations, and a possibility of fetal hypoxia. Even so, 5 to 10 scalp pH measurements would be highly unusual.Postterm clients generally do not have a decreased amount of amniotic fluid (oligohydramnios).

A nurse is monitoring a full-term laboring client on oxytocin. What interventions would be priorities for the nurse if the client experiences late decelerations? Select all that apply. Give oxygen. Report the findings to the heath care professional (HCP). Perform a vaginal examination for prolapsed cord. Position the client on her left side. Stop the infusion of oxytocin.

Give oxygen. Report the findings to the heath care professional (HCP). Position the client on her left side. Stop the infusion of oxytocin. Explanation: The presence of late decelerations indicates that the fetus is affected by placental insufficiency. There is decreased blood flow and oxygen available during the contraction when the uterine blood vessels are compressed during a contraction. Interventions to increase placental perfusion include positioning the client on her left side and stopping the infusion of oxytocin. Giving oxygen will reduce fetal hypoxia. Decelerations also need to be reported to the HCP immediately. Prolonged decelerations indicate cord compression. A vaginal exam would not be a priority at this time.

A client with eclampsia begins to experience a seizure. Which intervention should the nurse do immediately? Pad the side rails. Place a pillow under the left buttock. Insert a padded tongue blade into the mouth. Maintain a patent airway.

Maintain a patent airway. Explanation: The priority for the pregnant client having a seizure is to maintain a patent airway to ensure adequate oxygenation to the mother and the fetus. Additionally, oxygen may be administered by face mask to prevent fetal hypoxia.Because the client is diagnosed with eclampsia, she is at risk for seizures. Thus, seizure precautions, including padding the side rails, should have been instituted prior to the seizure.Placing a pillow under the client's left buttock would be of little help during a tonic-clonic seizure.Inserting a padded tongue blade is not recommended because injury to the client or nurse may occur during insertion attempts

A client with active genital herpes is admitted to the labor and birth unit during the first stage of labor. Which plan of care does the nurse anticipate for this client? Start an IV and give penicillin G every 4 hours until birth. Administer tocolytics as prescribed until the active lesions are healed. Administer valacyclovir 500 mg orally every 6 hours while in active labor. Prepare the client and partner for a cesarean birth as soon as possible.

Prepare the client and partner for a cesarean birth as soon as possible. Explanation: For a client with active genital herpes lesions, cesarean birth helps avoid infection transmission to the neonate, which would occur during a vaginal birth. Penicillin G is given for a bacterial colonization of group B streptococcus. Tocolytics are given to stop labor; they are not appropriate treatment. Valacyclovir is a treatment for an active herpes infection, but would not work in time for the client to deliver vaginally.

The nurse is caring for a postterm newborn. What interventions will the nurse include in the client's plan of care? Select all that apply. Encourage early feedings. Assess for respiratory distress. Assess for hypoglycemia. Double-wrap the infant in blankets. Examine the indirect Coombs test.

Encourage early feedings. Assess for respiratory distress. Assess for hypoglycemia. Double-wrap the infant in blankets. Explanation: Postterm newborns are at risk for hypoglycemia, meconium aspiration, and hypothermia, so the nurse should assess for all these disorders. Respiratory distress can occur after meconium aspiration, so the infant should be monitored closely for increased respiratory rates, grunting, retractions, and nasal flaring. Encouraging early feedings helps prevent hypoglycemia. Double-wrapping infants in blankets after they have been removed from the radiant warmer is done to prevent hypothermia. An indirect Coombs test would be related to jaundice.

The nurse is receiving shift report on four clients on an antenatal unit. The four clients are: (1) a 35-week-gestation mother with severe preeclampsia started on a maintenance dose of magnesium sulfate 1 hour ago; (2) a 30-week-gestation client with preterm labor on an oral tocolytic and having no contractions in 6 hours; (3) a hyperemesis client with emesis 4 times in the past 12 hours; and (4) a 33-week-gestation client with placenta previa who began to feel pelvic pressure during change of shift report. Which action should the nurse take first? Evaluate the client with preeclampsia for maternal and fetal tolerance of magnesium sulfate and the labor pattern. Assess the client with preterm labor for tolerance of tocolytics and the labor pattern. Assess the hyperemesis client for nausea for further emesis, or dehydration. Evaluate the placenta previa client without an exam.

Evaluate the placenta previa client without an exam. Explanation: The first action taken should be to evaluate the placenta previa client who has pelvic pressure. The pelvic pressure may be caused by a fetal head creating pressure in the pelvis indicating a potential birth. This client should be evaluated without a pelvic exam and then consult with the health care provider (HCP). A vaginal exam is contraindicated as it may stimulate bleeding of the placenta. The second action would be to complete an assessment on the client with preeclampsia and her fetus to evaluate for tolerance and effectiveness of the magnesium sulfate. The hyperemesis client needs to be evaluated for hydration status and for medication. The preterm labor client is stable on the oral medication and should be seen last.

A client is admitted to the labor and delivery unit in labor with blood flowing down her legs. What would be the priority nursing intervention? Place an indwelling catheter. Monitor fetal heart tones. Perform a cervical examination. Prepare the client for cesarean birth.

Monitor fetal heart tones. Explanation: Monitoring fetal heart tones would be the priority, due to a possible placenta previa or abruptio placentae. Although an indwelling catheter may be placed, it is not a priority intervention. Performing a cervical examination would be contraindicated because any agitation of the cervix with a previa can result in hemorrhage and death for the mother or fetus. Preparing the client for a cesarean birth may not be indicated. A sonogram will need to be performed to determine the cause of bleeding. If the diagnosis is a partial placenta previa, the client may still be able to deliver vaginally.

A client, 7 months pregnant, is admitted to the unit with abdominal pain and bright red vaginal bleeding. Which action should the nurse take? Place the client on her left side and start supplemental oxygen, as ordered. Administer I.V. oxytocin, as ordered. Ease the client's anxiety by assuring her that everything will be all right. Massage the client's fundus.

Place the client on her left side and start supplemental oxygen, as ordered. Explanation: The client's signs and symptoms indicate abruptio placentae, which decreases fetal oxygenation. To maximize fetal oxygenation, the nurse should place the client on her left side to increase placental blood flow to the fetus and administer supplemental oxygen, as ordered, to increase the blood oxygen level. Administering oxytocin is not appropriate because this drug stimulates contractions, which further reduce fetal oxygenation. The nurse cannot assure the client that everything will be all right, only that everything possible will be done to help her and her fetus. Fundal massage is used only during the postpartum period to control hemorrhage.

A client's partner uses the call bell to tell the nurse that the client's membranes have ruptured and "something is hanging out on the bed!" The nurse visualizes an overt prolapsed umbilical cord. What is the priority nursing action? Place the mother in a knee-to-chest position. Palpate the cord for pulsations before notifying the physician. Attempt an external cephalic rotation. Restore circulation by stimulating the cord with a sterile glove.

Place the mother in a knee-to-chest position. Explanation: The knee-to-chest position helps lift the presenting part off the umbilical cord. If, upon vaginal examination, a loop of cord is discovered, the nurse should keep gloved fingers in the vagina and push on the fetal presenting part to keep the part off the cord, thus relieving cord compression until the physician or midwife arrives. It is inappropriate to attempt an external cephalic rotation. Cord pulsations may not be felt; therefore, oxygen should be administered and electronic fetal monitoring should be put in place immediately to monitor the fetal heart rate and well being.

The nurse caring for the laboring client performs a sterile vaginal exam. Exam results are dilated 10 cm, effaced 100%, and +2 station. What is the priority nursing intervention? Initiate oxytocin infusion. Call anesthesia to give epidural anesthesia. Prepare for birth of the neonate. Assess for rupture of the membranes.

Prepare for birth of the neonate. Explanation: A client who is fully dilated is about to begin pushing. Appropriate actions for this time include assessing vital signs every 15 minutes, positioning for effective pushing, and preparing for delivery. Oxytocin is administered to induce labor or to help the uterus contract after birth; it would be inappropriate to administer to a client entering the second stage of labor. It is inappropriate to insert an epidural when the client is ready to start pushing. Status of membranes would have been determined during the sterile vaginal exam.

A 19-year-old primigravid client is being discharged home after hospitalization for hyperemesis gravidarum and is being referred to home health care. The nurse should develop a discharge plan that includes which interventions? Select all that apply. Refer client to a nutritionist for the following day. Ensure that the client has a prescription for an antiemetic. Ask the health care provider for an anxiolytic prescription. Encourage return to normal routine when client feels ready. Coordinate follow-up appointment with provider in 6 weeks. Discuss plan of care and discharge instructions with client. Refer client to a nutritionist for the following day. Ensure that the client has a prescription for an antiemetic. Ask the health care provider for an anxiolytic prescription. Encourage return to normal routine when client feels ready. Coordinate follow-up appointment with provider in 6 weeks. Discuss plan of care and discharge instructions with client

Refer client to a nutritionist for the following day. Ensure that the client has a prescription for an antiemetic. Encourage return to normal routine when client feels ready. Discuss plan of care and discharge instructions with client. The nurse case manager should refer the client to a nutritionist so the client is aware of and can be monitored regarding her food intake to assure transition to a normal pregnancy diet with intake of adequate nutrients to support growth and development of the fetus. A PRN (as needed) prescription for an antiemetic is useful to overcome occasional episodes of nausea and vomiting. Encouraging a return to normal activities when the client feels ready gives the client a goal to look forward to and activity is not contraindicated in hyperemesis when the client feels ready to initiate activity. Discussion of the plan of care and discharge instructions is a standard of care when discharging a client from a health care facility. There is no indication for an anxiolytic, and hyperemesis gravidarum typically is not associated with anxiety. Six weeks is too long to wait for a follow-up appointment post hospitalization.

A 27-year-old primigravid client with insulin-dependent diabetes at 34 weeks' gestation undergoes a nonstress test, the results of which are documented as reactive. What should the nurse tell the client that the test results indicate? A contraction stress test is necessary. The nonstress test should be repeated. Chorionic villus sampling is necessary. There is evidence of fetal well-being.

There is evidence of fetal well-being. Explanation: The nonstress test is considered reactive when two or more fetal heart rate accelerations of at least 15 bpm occur (from a baseline fetal heart rate of 120 to 160 bpm), along with fetal movement, during a 10- to 20-minute period. A reactive nonstress test indicates fetal heart rate accelerations and well-being. There is no indication for further evaluation (such as a contraction stress test). However, contraction stress tests are commonly scheduled for pregnant clients with insulin-dependent diabetes in the latter part of pregnancy and are repeated periodically until birth. Chorionic villus sampling is usually performed early in the pregnancy to detect fetal abnormalities.

Several pregnant clients are waiting to be seen in the triage area of the obstetrical unit. Which client should the nurse see first? a client at 13 weeks' gestation who is experiencing nausea and vomiting three times a day with + 1 ketones in her urine a client at 37 weeks' gestation who is an insulin-dependent diabetic and experiencing 3 to 4 fetal movements per day a client at 32 weeks' gestation who has preeclampsia and +3 proteinuria and who is returning for evaluation of epigastric pain a primigravida at 17 weeks' gestation who reports not feeling fetal movement at this point in her pregnancy

a client at 32 weeks' gestation who has preeclampsia and +3 proteinuria and who is returning for evaluation of epigastric pain Explanation: A preeclamptic client with +3 proteinuria and epigastric pain is at risk for seizing, which would jeopardize the mother and the fetus. Thus, this client would be the highest priority. The client at 13 weeks' gestation with nausea and vomiting is a concern because the presence of ketones indicates that her body does not have glucose to break down. However, this situation is a lower priority than the preeclamptic client or the insulin-dependent diabetic. The insulin-dependent diabetic is a high priority; however, fetal movement indicates that the fetus is alive but may be ill. As few as four fetal movements in 12 hours can be considered normal. (The client may need additional testing to further evaluate fetal well-being.) The client who is at 17 weeks' gestation may be too early in her pregnancy to experience fetal movement and would be the last person to be seen.

A client at 7 weeks' gestation is being treated for a ruptured ectopic pregnancy in the emergency department. The client's vital signs are blood pressure 84/45 mmHg, heart rate 122 beats/min, respiratory rate 26 breaths/min, temperature 98.6°F (36.8°C). What is the nurse's priority? administering fluid resuscitation and preparing the client for surgery administering analgesics and teaching the client about methotrexate monitoring vaginal discharge and performing focused abdominal assessment ensuring strict infection control and providing psychosocial support

administering fluid resuscitation and preparing the client for surgery Explanation: Most of the actions listed with the exception of teaching about methotrexate are interventions that may be performed by the nurse caring for a client with a ruptured ectopic pregnancy. The fact that the rupture has already occurred means methotrexate is contraindicated. The nurse should monitor vaginal losses, assess the client's abdomen, treat her pain, and prevent infection while providing psychosocial support. However, the priority is maintaining perfusion pressure and ensuring swift surgical intervention to correct this life-threatening medical emergency. When an ectopic pregnancy ruptures, the client experiences intra-abdominal bleeding. The fact that the client's blood pressure, heart rate, and respiratory rate all reflect hemodynamic instability means the nurse's priority is maintenance of perfusion through fluid resuscitation and correcting the blood loss through the surgical intervention.

Rho (D) immune globulin (RhoGAM) is prescribed for a client before she is discharged after a spontaneous abortion. The nurse instructs the client that this drug is used to prevent which condition? development of a future Rh-positive fetus an antibody response to Rh-negative blood a future pregnancy resulting in abortion development of Rh-positive antibodies

development of Rh-positive antibodies Explanation: Rh sensitization can be prevented by Rho(D) immune globulin, which clears the maternal circulation of Rh-positive cells before sensitization can occur, thereby blocking maternal antibody production to Rh-positive cells. Administration of this drug will not prevent future Rh-positive fetuses, nor will it prevent future abortions. An antibody response will not occur to Rh-negative cells. Rh-negative mothers do not develop sensitivities if the fetus is also Rh negative.

On arrival at the emergency department, a client tells the nurse that she suspects that she may be pregnant but has been having a small amount of bleeding and has severe pain in the lower abdomen. The client's blood pressure is 70/50 mm Hg, and her pulse rate is 120 bpm. The nurse notifies the primary health care provider (HCP) immediately because of the possibility of which complication? ectopic pregnancy. abruptio placentae. gestational trophoblastic disease. complete abortion.

ectopic pregnancy. Explanation: The client's signs and symptoms indicate a probable ectopic pregnancy, which can be confirmed by ultrasound examination or by culdocentesis. The HCP is notified immediately because hypovolemic shock may develop without external bleeding. Once the fallopian tube ruptures, blood will enter the pelvic cavity, resulting in shock. Abruptio placentae would be manifested by a board-like uterus in the third trimester. Gestational trophoblastic disease would be suspected if the client exhibited no fetal heart rate and symptoms of pregnancy-induced hypertension before 20 weeks' gestation. A client with a complete abortion would exhibit a normal pulse and blood pressure with scant vaginal bleeding.

A 36-year-old multigravid client is admitted to the hospital with possible ruptured ectopic pregnancy. When obtaining the client's history, which finding would be most important to identify as a predisposing factor? urinary tract infection marijuana use during pregnancy episodes of pelvic inflammatory disease use of estrogen-progestin contraceptives

episodes of pelvic inflammatory disease Explanation: Anything that causes a narrowing or constriction in the fallopian tubes so that a fertilized ovum cannot be properly transported to the uterus for implantation predisposes an ectopic pregnancy. Pelvic inflammatory disease is the most common cause of constricted or narrow tubes. Developmental defects are other possible causes. Ectopic pregnancy is not related to urinary tract infections. Use of marijuana during pregnancy is not associated with ectopic pregnancy, but its use can result in cognitive reduction if the mother's use during pregnancy is extensive. Progestin-only contraceptives and intrauterine devices have been associated with ectopic pregnancy.

Before surgery to remove an ectopic pregnancy and the fallopian tube, which sign or symptom would alert the nurse to the possibility of tubal rupture? amount of vaginal bleeding and discharge falling hematocrit and hemoglobin levels slow, bounding pulse rate of 80 bpm marked abdominal edema

falling hematocrit and hemoglobin levels Explanation: Diaphoresis, or profuse sweating, indicates shock, which occurs if the tube ruptures. Other common symptoms of tubal rupture include severe knife-like lower quadrant abdominal pain, referred shoulder pain, and falling blood pressure. The amount of vaginal bleeding that is evident is a poor estimate of actual blood loss. Slight vaginal bleeding, commonly described as spotting, is common. A rapid, thready pulse, a symptom of shock, is more common with tubal rupture than a slow, bounding pulse. Abdominal edema is a late sign of a tubal rupture in ectopic pregnancy.

After the nurse reviews the primary health care provider's (HCP's) explanation of amniocentesis with a multigravid client, which complication, if stated by the client, indicates that she needs more teaching about the procedure? risk of infection possible miscarriage risk of club foot fetal organ malformations

fetal organ malformations Explanation: There is little risk for fetal organ malformations from amniocentesis. One of the primary risks of amniocentesis is stimulation of the uterus and possible miscarriage. Other risks include hemorrhage from penetration of the placenta, infection of the amniotic fluid, and puncture of the fetus. Club foot has been associated with amniocentesis, especially when it is performed before 15 weeks.

A client who's 3 months pregnant with her first child reports increasing morning sickness for the past month. Nursing assessment reveals a fundal height of 20 cm and no audible fetal heart tones. The nurse should suspect which complication of pregnancy? fetal demise ectopic pregnancy hyperemesis gravidarum gestational trophoblastic disease

gestational trophoblastic disease Explanation: Gestational trophoblastic disease causes increased nausea and vomiting, uterine enlargement beyond that expected for the number of weeks' gestation, absence of fetal heart tones, and vaginal spotting. Because the client exhibits most of these signs, she requires further evaluation. In fetal demise, uterine size decreases; the client's fundal height of 20 cm at 3 months' gestation is too large to indicate fetal demise. Absence of fetal heart tones is a sign of ectopic pregnancy; however, a fundal height of 20 cm doesn't support that diagnosis. Although hyperemesis gravidarum causes increased nausea and vomiting, the client's enlarged uterus suggests a different problem.

The client gives birth to a neonate who is given a score of 9 at 5 minutes on the Apgar rating system. How does the nurse interpret the neonate's physical condition? good fair poor critical

good Explanation: The Apgar rating system evaluates the neonate on the basis of heart rate, respiratory effort, muscle tone, reflex irritability, and color at 1- and 5-minute intervals after birth. The neonate receives a score between 0 and 10. The higher the score, the better the neonate's condition. Scores between 7 and 10 indicate good status. An Apgar score between 4 and 6 indicates fair condition with a possible need for oxygen, suction, and stimulation.An Apgar score between 0 and 3 is indicative of poor neonatal status and a need for resuscitation. This status is also sometimes referred to as critical.

During the initial assessment of a laboring client, the nurse notes the following: blood pressure 160/110 mm Hg, pulse 88 beats/minute, respiratory rate 22 breaths/minute, reflexes +3/+4 with 2 beat clonus. Urine specimen reveals +3 protein, negative sugar and ketones. Based on these findings, a nurse should expect the client to have which complaints? headache, blurred vision, and facial and extremity swelling abdominal pain, urinary frequency, and pedal edema diaphoresis, nystagmus, and dizziness lethargy, chest pain, and shortness of breath

headache, blurred vision, and facial and extremity swelling Explanation: The client is exhibiting signs of preeclampsia. In addition to hypertension and hyperreflexia, most clients with preeclampsia have edema. Headache and blurred vision are indications of the effects of the hypertension. Abdominal pain, urinary frequency, diaphoresis, nystagmus, dizziness, lethargy, chest pain, and shortness of breath are inconsistent with a diagnosis of preeclampsia.

What conditions would the nurse expect to find in in a preterm neonate suffering from cold stress? yellowish undercast to the skin color increased abdominal girth hyperactivity and twitching slowed respirations

hyperactivity and twitching Explanation: A neonate with cold stress must produce heat through increased metabolism, causing oxygen use to increase and glycogen stores to be quickly depleted leading to hypoglycemia. Hyperactivity and twitching are signs of hypoglycemia.Yellowish undercast to the skin color suggests jaundice related to excessive bilirubin levels, not cold stress.Increased abdominal girth suggests abdominal distention, possibly indicating necrotizing enterocolitis. It is unrelated to cold stress or possible hypoglycemia.Increased, not slowed, respirations are associated with neonatal cold stress and hypoglycemia.

A client in her first trimester of pregnancy comes to the prenatal clinic and states, "I feel nauseous and I'm vomiting all the time. I can't even keep down water." This client should be evaluated for what condition? morning sickness eclampsia hyperemesis gravidarum hydramnios

hyperemesis gravidarum Explanation: Hyperemesis gravidarum differs from the nausea and vomiting (morning sickness) that normally occur during pregnancy. It's characterized by excessive vomiting that can lead to dehydration and starvation. Without treatment, metabolic changes can lead to severe complications, even death, of the fetus or mother. Eclampsia is the most serious form of gestational hypertension. It's characterized by hypertension, seizures, coma, edema, and proteinuria. Hydramnios is an overproduction of amniotic fluid that causes uterine distension.

The nurse should be especially alert for what problem when caring for a term neonate, who weighed 10 lb (4,500 g) at birth, 1 hour after a vaginal birth? hypoglycemia hypercalcemia hypermagnesemia hyperbilirubinemia

hypoglycemia Explanation: The neonate would be considered large for gestational age (LGA) because the neonate weighs more than 4,000 g (90th percentile). Therefore, the nurse needs to assess for the possibility of complications. Hypoglycemia is a problem for the LGA neonate because glycogen stores are quickly used to maintain the weight. Other common complications for an LGA neonate include hyperbilirubinemia from the bruising and polycythemia, cephalohematoma, caput succedaneum, molding, phrenic nerve paralysis, and a fractured clavicle. However, hyperbilirubinemia would not be evident 1 hour after birth. Hypercalcemia is not usually found in the LGA neonate. Hypocalcemia is common in infants of diabetic mothers. Hypermagnesemia may occur in neonates whose mothers received large doses of magnesium sulfate to treat severe preeclampsia

A neonate is admitted to the neonatal intensive care unit for observation with a diagnosis of probable meconium aspiration syndrome (MAS). The neonate weighs 10 lb, 4 oz (4,650 g) and is at 41 weeks' gestation. What would be the priority problem for this neonate? impaired skin integrity hyperglycemia risk for impaired patent-infant-child attachment impaired gas exchange

impaired gas exchange Explanation: The priority problem for the neonate with probable MAS is impaired gas exchange related to the effects of respiratory distress. Obstruction of the airways may be complete or partial. Meconium aspiration may lead to pneumonia or pneumothorax. Establishing adequate respirations is the primary goal. Impaired skin integrity is a concern, but establishing and maintaining an airway and gas exchange is always the priority. Hypoglycemia tends to be a problem for large-for-gestational-age babies, not hyperglycemia. If the parents do not express interest or concern for the neonate, then risk for impaired parent-infant-child attachment may be appropriate once the airway is established.

When teaching a group of pregnant adolescent clients about reproduction and conception, the nurse is correct when stating that fertilization occurs: in the uterus. when the ovum is released. near the fimbriated end. in the first third of the fallopian tube.

in the first third of the fallopian tube. Explanation: Fertilization occurs in the first third of the fallopian tube. After ovulation, an ovum is released by the ovary into the abdominopelvic cavity. It enters the fallopian tube at the fimbriated end and moves through the tube on the way to the uterus. Sperm cells "swim up" the tube and meet the ovum in the first third of the fallopian tube. The fertilized ovum then travels to the uterus and implants. Nurses must know where fertilization occurs because of the risk of an ectopic pregnancy.

A multigravid client in active labor at 39 weeks' gestation has a history of smoking one to two packs of cigarettes daily. Which problem is the nurse most likely to find during the infant's assessment? sedation hyperbilirubinemia low birth weight hypocalcemia

low birth weight Explanation: Neonates born to mothers who smoke tend to have lower-than-average birth weights. Neonates born to mothers who smoke also are at higher risk for stillbirth, sudden infant death syndrome, bronchitis, allergies, delayed growth and development, and polycythemia. Maternal smoking is not related to higher neonatal sedation, hyperbilirubinemia, or hypocalcemia. Smoking may cause irritability, not sedation. Hyperbilirubinemia is associated with Rh or ABO incompatibility or the administration of intravenous oxytocin during labor. Approximately 50% of neonates born to mothers with insulin-dependent diabetes experience hypocalcemia during the first 3 days of life.

A multigravid client is admitted to the hospital with a diagnosis of ectopic pregnancy. The nurse anticipates that, because the client's fallopian tube has not yet ruptured, which medication may be prescribed? progestin contraceptives medroxyprogesterone methotrexate dyphylline

methotrexate Explanation: Because the fallopian tube has not yet ruptured, methotrexate may be given, followed by leucovorin. This chemotherapeutic agent attacks the fast-growing zygote and trophoblast cells. RU-486 is also effective. A hysterosalpingogram is usually performed after chemotherapy to determine whether the tube is still patent. Progestin-only contraceptives and medroxyprogesterone are ineffective in clearing the fallopian tube. Dyphylline is a bronchodilator and is not used.

A client has been in early labor, with contractions every 10 to 12 minutes, for the past 12 hours with no progression. What medication should the nurse anticipate to help stimulate this client's uterine contractions? estrogen magnesium sulfate oxytocin terbutaline

oxytocin Explanation: Oxytocin is the hormone responsible for stimulating uterine contractions. Pitocin, the synthetic form, has similar action, and may be given to some clients to induce or augment uterine contractions. Although estrogen plays a role in uterine contractions, it is not given in synthetic form to help uterine contractility. Progesterone has a relaxing effect on the uterus. Magnesium sulfate is used for maternal preeclampsia. It is not a tocolytic agent.

When the nurse is assessing a 34-year-old multigravid client at 34 weeks' gestation experiencing moderate vaginal bleeding, which symptom would most likely alert the nurse that placenta previa is present? painless vaginal bleeding uterine tetany intermittent pain with spotting dull lower back pain

painless vaginal bleeding Explanation: The most common assessment finding associated with placenta previa is painless vaginal bleeding. With placenta previa, the placenta is abnormally implanted, covering a portion or all of the cervical os. Uterine tetany, intermittent pain with spotting, and dull lower back pain are not associated with placenta previa. Uterine tetany is associated with oxytocin administration. Intermittent pain with spotting commonly is associated with a spontaneous abortion. Dull lower back pain is commonly associated with poor maternal posture or a urinary tract infection with renal involvement.

A primigravid client is experiencing a prolonged second stage of labor with a fetus suspected of weighing more than 4 kg. Which intervention is most important? preparing for a vacuum-assisted delivery administering an IV fluid bolus preparing for an emergency cesarean birth performing the McRoberts maneuver

performing the McRoberts maneuver Explanation: A prolonged second stage of labor with a large fetus could indicate a shoulder dystocia at birth. Immediate nursing actions for a shoulder dystocia include suprapubic pressure and the McRoberts maneuver. If after interventions for vaginal birth with a shoulder dystocia fail, an emergency cesarean birth may be needed but is not indicated at this time. A vacuum-assisted birth would be contraindicated due to increased risk of shoulder dystocia with a macrosomic infant. An IV fluid bolus may be indicated for fetal distress, but there is not enough information to establish that they are needed at this time.

A nurse is completing a physical assessment of a neonate following birth. When completing the musculoskeletal assessment, which findings would indicate developmental dysplasia of the hip (DDH)? Select all that apply. negative Ortolani test positive Barlow test asymmetrical leg skin folds limitation in adduction of the affected leg lengthening of the affected leg

positive Barlow test asymmetrical leg skin folds Explanation: Developmental dysplasia (dislocation) of the hip is an abnormal formation of the hip joint in which the ball on the top of the femur is not held firmly in the socket. A neonate with DDH will have a positive Ortolani test, a positive Barlow test, and asymmetrical skin folds in the thigh. The affected leg has limited abduction and appears shorter than the unaffected leg in a neonate with DDH.

The nurse is caring for a woman who gave birth vaginally to a healthy 6 pound (2.72 kg) newborn after a 2-hour labor at 37 weeks gestation. For which complication will the nurse assess as a priority due to the increased risk in this client? postpartum infection delay in lactation postpartum hemorrhage delayed infant bonding

postpartum hemorrhage Explanation: The client's labor was under 3 hours in length, which meets the definition for precipitous labor. This increases the risk for postpartum hemorrhage but decreases the risk for infection. The client is at early term (37 weeks) and gave birth vaginally without any noted complications, so there should be no delay in either lactation or infant bonding.

The nurse covers the myelomeningocele of a neonate with a sterile dressing. Which statements direct the nurse's action? conserving energy preventing infection promoting neural tube sac drainage conserving body heat

preventing infection Explanation: The nurse needs to provide special care to the neural tube sac to prevent infection. Allowing the sac to dry could result in cracks that allow microorganisms to enter. Pressure on the sac could cause it to rupture creating a portal of entry for microorganisms. Administering antibiotics and keeping the sac free from urine and stool are other measures to prevent infection. Conserving energy is not a concern for neonates, including those with a myelomeningocele. Like all neonates, a neonate with a myelomeningocele must be kept warm, but care must be taken to avoid drying out the neural tube sac with a radiant heater or exerting pressure using a sheet or blanket over the sac.

A 32-year-old multigravida returns to the clinic for a routine prenatal visit at 36 weeks' gestation. The assessments during this visit include: blood pressure 140/90 mm Hg; pulse 80 beats/min; respiratory rate 16 breaths/min. What further information should the nurse obtain to determine if this client is becoming preeclamptic? headaches blood glucose level proteinuria peripheral edema

proteinuria xplanation: The two major defining characteristics of preeclampsia are blood pressure elevation of 140/90 mm Hg or greater and proteinuria. Because the client's blood pressure meets the gestational hypertension criteria, the next nursing responsibility is to determine if she has protein in her urine. If she does not, then she may be having transient hypertension. The peripheral edema is within normal limits for someone at this gestational age, particularly because it is in the lower extremities. While, the preeclamptic client may significant edema in the face and hands, edema can be caused by other factors and is not part of the diagnostic criteria. Headaches are significant in pregnancy-induced hypertension but may have other etiologies. The client's blood glucose level has no bearing on a preeclampsia diagnosis.

The nurse is reviewing a pregnant client's immunization record. Which immunizations are contraindicated during pregnancy and are not updated at this time? Select all that apply. tetanus rubella mumps chickenpox live attenuated influenza vaccine (LAIV) hepatitis B

rubella mumps chickenpox live attenuated influenza vaccine (LAIV) Explanation: During pregnancy, live virus immunizations are not administered because of potential teratogenicity. These immunizations include chickenpox, rubella, mumps, small pox, and live attenuated influenza vaccine. Vaccines containing killed viruses (tetanus, diphtheria, hepatitis B) may be administered, and in fact are recommended by the CDC if the immunizations are not up to date.

The nurse is performing a routine assessment of a 37-week pregnant client. What should the nurse implement during this prenatal visit? screening for group B strep and STIs administration of rubeola vaccine education about hospital registration administration of RhoGAM if the mother is Rh-negative

screening for group B strep and STIs Explanation: During each prenatal visit the mother's blood pressure, weight, and urine are assessed and the fundal height is measured. Between 37 and 40 weeks, screening for group B streptococcus, gonorrhea, and chlamydia is also done. The mother should have already preregistered at the hospital. RhoGAM is given at 28 weeks. The measles vaccine is given postpartum to the mother who is not immunized.

An 18-year-old pregnant client tells the nurse that she is concerned that she may not be able to take care of herself during her pregnancy. She states that she is not sure what prenatal care is available, or if she should access it. The nurse should recognize that the client: may not take care of herself. may not be fit to take care of a child. needs to find a second job. should be referred to community resources available for pregnant women.

should be referred to community resources available for pregnant women. Explanation: The client needs to know that many freely available resources exist, and the nurse should help her to find such resources. It doesn't necessarily mean that the client has no interest in caring for herself or her child.

The nurse is admitting a newborn to the nursery. Report reveals that the newborn was slow to crown and delivery of the head and chin was difficult. For which complication would the nurse need to assess? shoulder dystocia immature lung function hypoglycemia small birth weight

shoulder dystocia Explanation: This neonate exhibits findings of a post-term infant. Typically they are larger in size and more at risk for having shoulder dystocia. Immature lung function, hypoglycemia, and small birth weight are more common in pre-term infants.

The nurse instructs a primigravid client to increase her intake of foods high in magnesium. This nutrient plays a role in which process? prevention of demineralization of the mother's bones synthesis of proteins, nucleic acids, and fats amino acid metabolism synthesis of neural pathways in the fetus

synthesis of proteins, nucleic acids, and fats Explanation: Magnesium aids in the synthesis of protein, nucleic acids, and fats. It is important for cell growth and neuromuscular function. Magnesium also activates the enzymes for metabolism of protein and energy. Calcium prevents demineralization of the mother's bones. Vitamin B6 is important for amino acid metabolism. Folic acid assists in the development of neural pathways in the fetus.

The nurse is administering intravenous magnesium sulfate as prescribed for a client at 34 weeks' gestation with severe preeclampsia. What are desired outcomes of this therapy? Select all that apply. temperature, 98° F (36.7° C); pulse, 72 beats/min; respiratory rate, 14 breaths/min urinary output less than 30 mL/h fetal heart rate with late decelerations blood pressure less than 140/90 mm Hg deep tendon reflexes 2+ magnesium level = 5.6 mg/dL (2.8 mmol/L)

temperature, 98° F (36.7° C); pulse, 72 beats/min; respiratory rate, 14 breaths/min deep tendon reflexes 2+ magnesium level = 5.6 mg/dL (2.8 mmol/L) Explanation: The use of magnesium sulfate as an anticonvulsant acts to depress the central nervous system by blocking peripheral neuromuscular transmissions and decreasing the amount of acetylcholine liberated. The primary goal of magnesium sulfate therapy is to prevent seizures. While being used, the temperature and pulse of the client should remain within normal limits. The respiratory rate needs to be greater than 12 respirations per minute (rpm). Rates at 12 rpm or lower are associated with respiratory depression and are seen with magnesium toxicity. Renal compromise is identified with a urinary output of less than 30 mL/hour. A fetal heart rate that is maintained within the 112 to 160 range is desired without later or variable decelerations. While extreme elevations of blood pressure must be treated, achieving a normal pressure carries the risk of decreasing perfusion to the fetus. Deep tendon reflexes should not be diminished or exaggerated. The therapeutic magnesium sulfate level of 5 to 8 mg/dL (2.5 to 4 mmol/L) is to be maintained.

During the first hour after a precipitous birth, the nurse should monitor a multiparous client for signs and symptoms of which complication? postpartum "blues" uterine atony intrauterine infection urinary tract infection

uterine atony Explanation: Because birth occurs so rapidly and the fetus is propelled quickly through the birth canal, the major complication of a precipitous birth is a boggy fundus, or uterine atony. The neonate should be put to the breast, if the mother permits, to allow for the release of natural oxytocin. In a hospital setting, the health care provider (HCP) will probably prescribe administration of oxytocin. The nurse should gently massage the fundus to ensure that it is firm. There is no relationship between a precipitous birth and postpartum "blues" or intrauterine infection. Postpartum "blues" usually do not occur until about 3 days postpartum, and symptoms of postpartum infection usually occur after the first 24 hours. There is no relationship between a precipitous birth and urinary tract infection even though the birth has been accomplished under clean rather than sterile technique. Symptoms of urinary tract infection typically begin on the first or second postpartum day.

During the fourth stage of labor, the client should be assessed carefully for uterine atony. complete cervical dilation. placental expulsion. umbilical cord prolapse.

uterine atony. Explanation: Uterine atony should be carefully assessed during the fourth stage. The second stage of labor begins with complete cervical dilation and ends with birth. The third stage begins immediately after birth and ends with the separation and expulsion of the placenta. Immediately after delivery, the placenta is evaluated carefully for completeness, and the client is assessed for excessive bleeding or a relaxed uterus. After delivery of the placenta is the fourth stage and assessing for relaxed uterus helps determine uterine atony. Umbilical cord prolapse, displacement of the umbilical cord to a position at or below the fetus's presenting part, occurs most commonly when amniotic membranes rupture before fetal descent. The client should be assessed for a visible or palpable umbilical cord in the birth canal, violent fetal activity, or fetal bradycardia with variable deceleration during contractions. The presence of umbilical cord prolapse requires an emergency delivery.


Kaugnay na mga set ng pag-aaral

Voting, Political Parties, and Interest Groups

View Set

Journeys 5th Grade Lesson 3 COMPREHENSION

View Set

International Business Chapter 13 Multiple Choice

View Set

Quiz 1 - Chapter 1: Exploring Economics

View Set

Controlled Substance Prescriptions

View Set

Managerial Accounting Smartbook 6

View Set

Chapter 1 - Introduction to Managerial Accounting

View Set